Endocrine Question
Endocrine Question
QID: 72
Topic: Cushing's Syndrome
Subject: Medicine
Explanation:
Cushings syndrome is one of the secondary causes of hypertension. Therefore the
blood pressure would be high. Along with Cushings, the other causes include
Hyperaldosteronism, Aortic coarctation, Pheochromocytoma and Stenosis of renal
artery. Remember the mnemonic C.H.A.P.S. for these 5 causes of secondary
hypertension.
Question #2
QID: 623
Topic: Cushing's Syndrome
Subject: Medicine
a) CT of abdomen
b) DST (dexamethasone supression test)
c) Random serum cortisol
d) MRI of brain
Explanation:
Cushing's syndrome is a constellation of clinical abnormalities caused by chronic high
blood levels of cortisol or related corticosteroids. Cushing's disease is Cushing's
syndrome that results from excess pituitary production of ACTH, usually secondary to
a pituitary adenoma. Typical symptoms include moon facies and truncal obesity with
thin arms and legs. Diagnosis is by history of receiving corticosteroids or by elevated
serum cortisol.
Question #3
QID: 981
Topic: Cushing's Syndrome
Subject: Medicine
A 52 year old woman, recently diagnosed with hypertension, presents to her family
physician complaining of hirsutism and acne. Laboratory examination reveals:
cortisol 1500 nmol/L (N: 200 - 660 nmol/L) aldosterone 200 pmol/L (N: 140 - 415
pmol/L) ACTH 4 pmol/L (N: 4 - 22 pmol/L)
a) Addison's disease
b) Cushing's syndrome
c) Bartter's syndrome
d) Renal artery stenosis
e) Conn's syndrome
Explanation:
In Cushing's syndrome, the level of corticosteroids is excessive, usually from
overproduction by the adrenal glands. Corticosteroids alter the amount and
distribution of body fat. Excessive fat develops throughout the torso and may be
particularly noticeable at the top of the back. A person with Cushing's syndrome
usually has a large, round face (moon face). The arms and legs are usually slender in
proportion to the thickened trunk. Muscles lose their bulk, leading to weakness.
High corticosteroid levels over time raise the blood pressure, weaken bones
(osteoporosis), and diminish resistance to infections. The risk of developing kidney
stones and diabetes is increased, and mental disturbances, including depression and
hallucinations, may occur. Women usually have an irregular menstrual cycle.
Children with Cushing's syndrome grow slowly and remain short. In some people, the
adrenal glands also produce large amounts of androgens (testosterone and similar
hormones), leading to increased facial and body hair in women and balding.
When doctors suspect Cushing's syndrome, they measure the level of cortisol, the
main corticosteroid hormone, in the blood. Normally, cortisol levels are high in the
morning and lower late in the day. In people who have Cushing's syndrome, cortisol
levels are very high throughout the day.
Question #4
QID: 96
Topic: Pheochromocytoma
Subject: Medicine
Explanation:
This patient needs to be screened for pheocromocytoma. Pheochromocytoma is a
tumor of the adrenal gland which causes very high levels of the catecholamines
(epinephrine and norepinephrine) to be secreted into the bloodstream. This can lead to
many sympathetic nervous system symptoms like elevated blood pressure,
palpitations, anxiety, diaphoresis, headaches, weight loss. These symptoms happen is
spurts or paroxysms.
Diagnoses is made by measuring the level of the catecholamines and their breakdown
products or metabolites which are called metanephrines in a 24 hour urine collection.
Treatment involves medicines to control the blood pressure and surgery to remove the
tumor. (Metanephrine levels are considered the most sensitive and specific test for a
pheochromocytoma, while vanillylmandelic acid is the least specific test and has a
false-positive rate greater than 15%.)
Recall the MEN syndromes. MEN (Multiple endocrine neoplasia) has three types:
This patient could have MEN II if you consider his family history.
Question #5
QID: 390
Topic: Pheochromocytoma
Subject: Medicine
Explanation:
A pheochromocytoma is a catecholamine-secreting tumor of chromaffin cells
typically located in the adrenals. It causes persistent or paroxysmal hypertension. The
classic triad of symptoms in patients with a pheochromocytoma consists of episodic
headache, diaphoresis, and tachycardia. Other common symptoms and signs include
postural hypotension, cold and clammy skin, severe headache, angina, palpitations,
nausea, vomiting, epigastric pain, visual disturbances, dyspnea, paresthesias,
constipation, and a sense of impending doom.
Question #6
QID: 918
Topic: Pheochromocytoma
Subject: Medicine
A 40-year-old woman has episodic headaches that last 30 minutes and are associated
with sweating, palpitations, and feelings of apprehension. She has lost 6.8 kg over the
past three months. Physical examination reveals a thin woman with a pulse of 112
beats/minute and a blood pressure of 150/100 mmHg lying and 130/80 mmHg
standing.
a) Pheochromocytoma
b) Hyperaldosteronism
c) Posterior fossa tumor
d) Renal artery stenosis
e) Carcinoid syndrome
Explanation:
A pheochromocytoma is a catecholamine-secreting tumor of chromaffin cells
typically located in the adrenals. It causes persistent or paroxysmal hypertension.
Diagnosis is by measuring catecholamine products in blood or urine. Imaging tests,
especially CT or MRI, help localize tumors. Treatment involves removal of the tumor
when possible. Drug therapy for control of BP includes alpha-blockade, possibly
combined with beta-blockade. Initiate a beta blocker only after adequate alpha
blockade (usually, 2 days).
Question #7
QID: 935
Topic: Pheochromocytoma
Subject: Medicine
A 40-year-old woman has episodic headaches that last 30 minutes and are associated
with sweating, palpitations, and feelings of apprehension. She has lost 6.8 kg over the
past three months. Physical examination reveals a thin woman with a pulse of 112
beats/minute and a blood pressure of 150/100 mmHg lying and 130/80 mmHg
standing.
Explanation:
A pheochromocytoma is a catecholamine-secreting tumor of chromaffin cells
typically located in the adrenals. It causes persistent or paroxysmal hypertension.
Diagnosis is by measuring catecholamine products in blood or urine. Imaging tests,
especially CT or MRI, help localize tumors. Treatment involves removal of the tumor
when possible. Drug therapy for control of BP includes α-blockade, possibly
combined with β-blockade.
Question #8
QID: 112
Topic: DM
Subject: Medicine
Which of the following diabetes drugs is contraindicated in liver and kidney failure?
a) Insulin
b) Metformin
c) Glyburide
d) Acarbose
Explanation:
Insulin works by directly pushing glucose from the bloodstream into the cells.
Metformin works by inhibiting hepatic gluconeogenesis. Glyburide is an oral
sulfonyl-urea hypoglycemic drug that increases pancreatic insulin secretion.
Metformin is contraindicated in people with any condition that could increase the risk
of lactic acidosis, including liver and kidney disorders (creatinine levels over 132
μmol/l).
Question #9
QID: 218
Topic: DM
Subject: Medicine
a) Acarbose
b) Metformin
c) Rosiglitazone
d) Glyburide
Explanation:
Metformin has been clearly established as the drug of choice in obese patients with
diabetes mellitus. Metformin is as effective as sulphonylureas at reducing HbA1c, and
most importantly has a beneficial effect on overall mortality in obese patients.
Metformin monotherapy is unlikely to be effective in patients who fail to respond to
sulphonylureas, but in patients who are secondary failures to sulphonylureas, the
addition of metformin causes substantial blood glucose lowering. Metformin should
be avoided in patients with renal dysfunction. Otherwise its effects on bodyweight,
serum lipids and its lack of hypoglycemia effect make it an excellent first line agent.
Question #10
QID: 317
Topic: DM
Subject: Medicine
Which of the following is the best test in the long term follow up of a diabetic patient?
a) HbA1c
b) C-Peptide
c) Fasting blood glucose
d) Random blood glucose
e) Diary of postprandial blood glucose values
Explanation:
There are two common ways to assess how well diabetes is controlled:
- frequent measurements of blood glucose, and
- measurement of glycohemoglobin (A1c).
Each method has its good and bad points, but combined they give a fairly accurate
picture of the state of glucose control in a diabetic. Most physicians will use both
methods.
Because red blood cells have life span of 120 days (3 months) measuring the level of
glycated hemoglobin in blood effectively shows the average blood glucose level for
the previous 6-8 weeks. In conjunction with regular blood glucose monitoring diaries,
A1c tests can help in determining if a patient's treatment plan needs altering in any
way to better control blood glucose levels. Studies have shown that glycohemoglobin
values in the "better ranges" correlate with less incidence of diabetic complications
later in life.
> A C-peptide test measures the level of this peptide in the blood. It is generally found
in amounts equal to insulin. A C-peptide test can be done when diabetes has just been
found and it is not clear whether type 1 diabetes or type 2 diabetes is present.
> Self-blood glucose monitoring is a valuable diabetes management tool, which
includes both random (pre- or post-prandial) and fasting blood sugar levels, as
recommended (not postprandial glucose levels only).
Question #11
QID: 797
Topic: DM
Subject: Medicine
Which one of the following is the best indicator of adequate control in a patient with
diabetes mellitus?
Explanation:
In the normal 120-day life span of the red blood cell, glucose molecules join
hemoglobin, forming glycated hemoglobin. In individuals with poorly controlled
diabetes, increases in the quantities of these glycated hemoglobins are noted.
The International Diabetes Federation recommend HbA1c values below 6.5%, while
the American Diabetes Association recommends that the HbA1c be below 7.0% for
most patients. A high HbA1c represents poor glucose control.
Persistent elevations in blood sugar (and therefore HbA1c) increase the risk for the
long-term vascular complications of diabetes such as coronary disease, heart attack,
stroke, heart failure, kidney failure, blindness, erectile dysfunction, neuropathy (loss
of sensation, especially in the feet), gangrene, and gastroparesis (slowed emptying of
the stomach).
Question #12
QID: 1103
Topic: DM
Subject: Medicine
A 34-year-old laborer comes to the office because of a 2 kg (5 lb) weight loss and an
increased appetite. He has diabetes mellitus and has been taking insulin in divided
doses. He says that home monitoring of his serum glucose concentration has shown
values from 15 mmol/L to 17.8 mmol/L.
Which of the following is the most appropriate management?
a) Add metformin
b) Change to another type of insulin
c) Increase his caloric intake
d) Increase his insulin dose
e) Redistribute his caloric intake
Explanation:
This patient has inadequately treated insulin-dependent diabetes. The goal serum
glucose for therapy is < 6.6 mmol/L. Since there is no indication that the insulin type
is incorrect (hypoglycemia), the patient simply requires an increased dosage of his
current insulin type.
Metformin is an oral antihyperglycemic agent that acts by increasing peripheral
utilization of glucose. The primary purpose of medications such as this is to delay or
avoid the requirement for exogenous insulin. Once a patient is on insulin, these drugs
are of little benefit. Secondarily, most patients with non-insulin dependent diabetes
have relative “insulin resistance” and it is for this reason that glucose utilization drugs
are employed as first line management.
This patient’s symptoms of increased weight loss and appetite are a function of his
poor glucose control, not of inadequate caloric intake. Poorly controlled insulin
dependent diabetics are often thin as they waste muscle to liberate glucose in an
attempt to provide glucose for the body. The defect is with the inability to utilize the
glucose already present so the end result is hyperglycemia and muscle wasting. For
similar reasons, there is no need to alter his caloric distribution.
Question #13
QID: 1554
Topic: DM
Subject: Medicine
A 75 year old white male with well-controlled type 2 diabetes mellitus is scheduled
for an abdominal CT scan with oral and intravenous iodinated contrast.
Which one of the following medications should be withheld 48 hours before and after
the procedure?
Explanation:
Metformin should be withheld before and after radiographic procedures with contrast,
due to its interaction with iodinated contrast materials. This interaction may cause
impaired renal function or lactic acidosis. The other drugs listed do not carry this risk.
Question #14
QID: 2676
Topic: DM
Subject: Medicine
Explanation:
The use of insulin therapy can be as appropriate in the older adult with diabetes as in
younger individuals. In patients with reduced muscle mass, such as the elderly
(especially those older than 80 years of age), using serum creatinine concentration to
estimate the glomeruler filtration rate may be misleading, and creatinine clearance
should be determined. If creatinine clearance is < 70 mL/min, metformin should not
be prescribed. The other two oral antidiabetic agents are safe to use in an elderly
patient without other initial laboratory data.
Question #15
QID: 2731
Topic: DM
Subject: Medicine
a) Osteoporosis
b) Hyperthyroidism
c) Polycycstic ovary syndrome
d) Right ventricular hypertrophy
e) Morbid truncal obesity
Explanation:
Recent data suggests that insulin resistance and hyperinsulinemia are important in the
pathogenesis of polycystic ovary syndrome (PCOS). Treatment with drugs that reduce
glucose levels, such as metformin, has been shown to correct many of the metabolic
abnormalities associated with PCOS. Such correction results in resumption of
ovulation, decreased insulin resistance, and improved beta-cell function; it also
produces improvement in cardiovascular risk factors such as dyslipidemia and
impaired fibrinolysis.
Question #16
QID: 2874
Topic: DM
Subject: Medicine
Explanation:
Question #17
QID: 2877
Topic: DM
Subject: Medicine
Explanation:
Thiazolidinediones decrease insulin resistance by binding to nuclear peroxisome
proliferators-activated receptors. They decrease gluconeogenesis and have no effect
on insulin release or production by the pancreas, or on intestinal carbohydrate
absorption.
Question #18
QID: 2953
Topic: DM
Subject: Medicine
A 45-year-old male was recently diagnosed with type 2 diabetes mellitus. He presents
to your office today to begin oral antihyperglycemic therapy. Results of a fasting
serum metabolic panel include the following:
Laboratory Findings
Sodium…………………………………………136
mmol/L (N 136-145)
Potassium…………………………………...…3.7
mmol/L (N 3.5-5.1)
Chloride………………………………………..102
mmol/L (N 98-107)
BUN………………………………………….…15
mg/dL (N 7-18)
Creatinine………………………………...……1.7
mg/dL (N 0.6-1.3)
Glucose………………………………………...183
mg/dL
a) Glipizide (Glucotrol)
b) Acarbose (Precose)
c) Metformin (Glucophage)
d) Pioglitazone (Actos)
Question #19
QID: 2970
Topic: DM
Subject: Medicine
Which one of her medications listed below is most likely to be linked to her new onset
of diabetes?
a) Atenolol (Tenormin)
b) Trazodone (Desyrel)
c) Paroxetine (Paxil)
d) Risperidone (Risperidal)
Explanation:
Recent data suggests a link between atypical antipsychotics and diabetes. In
particular, studies have suggested an increased risk of diabetes for patients on
risperidone, quetiapine, clozapine, and olanzapine. This association has been noted
even in patients who do not gain weight on the antipsychotic. The other medications
listed have not been implicated in diabetes.
Some experts recommend checking a fasting glucose level before and soon after
initiating an atypical antipsychotic. In addition, monitoring blood glucose every 3-6
months should be considered, especially in patients with other risk factors for
developing diabetes. Patients with preexisting diabetes may suffer worsened glycemic
control when started on an atypical antipsychotic.
Question #20
QID: 3155
Topic: DM
Subject: Medicine
a) Obesity
b) Insulin resistance
c) Renal insufficiency
d) Concurrent use of glyburide (DiaBeta, Micronase)
Explanation:
Metformin decreases glucose production in the liver and increases glucose uptake. It
has no effect on pancreatic insulin secretion. It is useful to help overcome insulin
resistance, and can be used concurrently with a sulfonylurea. Studies have proven its
safety and usefulness in the obese diabetic. It is contraindicated in renal insufficiency,
as increased concentrations of the drug can cause lactic acidosis. Other
contraindications include hepatic disease, a previous history of lactic acidosis, heart
failure treated with drugs, and chronic lung disease.
Question #21
QID: 3254
Topic: DM
Subject: Medicine
a) Pregnancy
b) Nonalcoholic fatty liver disease
c) Lipodystrophy associated with highly active antiretroviral therapy
d) Anemia
Explanation:
The thiazolidinediones rosiglitazone and pioglitazone are categorized as pregnancy
category C, due to experimental evidence of growth retardation in mid- to late
gestation in animal models. These drugs are approved for the treatment of type 2
diabetes mellitus, and may be beneficial in other conditions associated with insulin
resistance. Liver enzyme elevations are improved in patients with nonalcoholic fatty
liver disease. Although not very effective, the drugs are not contraindicated in patients
with lipodystrophy associated with highly active antiretroviral therapy. The anemia
associated with these drugs is mild and rarely causes clinical consequences.
Question #22
QID: 3289
Topic: DM
Subject: Medicine
A 45-year-old male comes to your office for follow-up of his diabetes mellitus, which
was diagnosed 3 months ago. He wanted to try lifestyle modification, including diet,
exercise, and weight loss, before trying medications. He has lost 3 kg (7 lb), but still is
14 kg (31 lb) overweight. His examination is unremarkable, as is his chemistry
profile. His hemoglobin A1c level is 8.3%.
a) Exenatide (Byetta)
b) Glyburide (Micronase, DiaBeta)
c) Insulin
d) Metformin (Glucophage)
e) Rosiglitazone (Avandia)
Explanation:
The Diabetes Association recommends lifestyle intervention along with metformin as
initial therapy for type 2 diabetes. If the follow-up hemoglobin A 1c is more than 7.0%,
then insulin (most effective), a sulfonylurea (least expensive), or a glitazone (no risk
of hypoglycemia) can be added. The hemoglobin A1c should be checked every 3
months until it is less than 7.0%, and every 6 months thereafter.
Question #23
QID: 3307
Topic: DM
Subject: Medicine
Which one of the following statements regarding blood glucose monitoring in patients
with type 1 diabetes is most correct?
Explanation:
According to the Diabetes Control and Complications Trial, patients should assess
fingertip blood glucose levels at least 3 times per day, including at bedtime, as well as
before and after exercising and before driving. Although testing from other sites is
usually reliable, it can be inaccurate if performed within 60 minutes of meals or after
exercise. If a patient’s random blood glucose level is less than 5.6 mmol/L (100
mg/dL), a small snack should be eaten. Preprandial results should be 5.0-7.3 mmol/L
(90-130 mg/dL), and postprandial levels should be less than 10.0 mmol/L (180
mg/dL). Continuous blood glucose monitoring has been shown to generally lead to
tighter control and lower hemoglobin A1c levels.
Question #24
QID: 3328
Topic: DM
Subject: Medicine
Which one of the following statements is true regarding the use of exenatide (Byetta)
in the treatment of diabetes?
Explanation:
Exenatide is indicated in combination with metformin, and/or a sulfonylurea to
improve glycemic control in patients with type 2 diabetes mellitus, when maximally
tolerated doses of these therapies in addition to diet and exercise do not provide
adequate glycaemic control.
In Sep 2012, Health Canada has approved exenatide injection added on to insulin
glargine (with or without metformin) to improve glycemic control.
It rarely causes hypoglycemia, except when used with sulfonylureas. It frequently
causes weight loss.
There is no experience with Byetta in children and adolescents less than 18 years and
therefore, use of Byetta is not recommended in this age group.
Question #25
QID: 5567
Topic: DM
Subject: Medicine
Explanation:
For geriatric patients in long-term care facilities, the predictable glucose control of
insulin glargine is the best approach to consider initially.
>The current guidelines do not recommend a strict diet for frail diabetic patients in
nursing homes.
>Exenatide is not recommended for the frail elderly because of concerns about weight
loss and nausea.
>Heart failure precludes the use of pioglitazone, and renal failure precludes the use of
metformin.
Question #26
QID: 5596
Topic: DM
Subject: Medicine
Which one of the following most increases insulin sensitivity in an overweight patient
with diabetes mellitus?
a) Metformin (Glucophage)
b) Acarbose (Precose)
c) Glyburide (DiaBeta, Micronase)
d) NPH insulin
Question #27
QID: 5623
Topic: DM
Subject: Medicine
A 40-year-old female comes to your office for a routine examination. She has been in
good health and has no complaints other than obesity. Her mother is diabetic and the
patient has had a child that weighed 4kg (9 lb) at birth. Her examination is negative
except for her obesity. A fasting glucose level is 7.1 mmol/L, and when repeated 2
days later it is 7.5 mmol/L. Which one of the following would be most appropriate at
this point?
a) Diagnose type 2 diabetes mellitus and begin diet and exercise therapy
b) Begin an oral hypoglycemic agent
c) Order a glucose tolerance test
d) Tell the patient that she has impaired glucose homeostasis but is not diabetic
Explanation:
The criteria for diagnosing diabetes mellitus include any one of the following:
symptoms of diabetes (polyuria, polydipsia, weight loss) plus a casual glucose level
11.1 mmol/L; a fasting plasma glucose level 7 mmol/L; or a 2-hour postprandial
glucose level 11.1 mmol/L after a 75-gram glucose load or a glycated hemoglobin
(A1C) value of ≥6.5%. In the absence of unequivocal hyperglycemia the test must be
repeated on a different day.
The criteria for impaired glucose homeostasis include either a fasting glucose level of
6.1-6.9 mmmol/L (impaired fasting glucose) or a 2-hour glucose level of 7.8-11
mmol/L on an oral glucose tolerance test or or an A1C of 6.0% to 6.4%. Normal
values are now considered 6.1 mmol/L for fasting glucose and 7.8 mmol/L for the 2-
hour glucose level on an oral glucose tolerance test.
Question #28
QID: 5657
Topic: DM
Subject: Medicine
Explanation:
This patient has prediabetes, which is defined as having a fasting plasma glucose level
of 5.6 to 6.9 mmol/L.
These patients are at high risk for developing diabetes mellitus later in life.
Prediabetes is associated with metabolic syndrome, and weight loss, exercise, and
certain pharmacologic agents have been shown to prevent or delay the subsequent
development of diabetes mellitus.
Question #29
QID: 6105
Topic: DM
Subject: Medicine
In patients with type 2 diabetes mellitus, intensive glycemic control has not been
shown to be beneficial for which one of the following diabetic complications?
a) Peripheral neuropathy
b) Foot infections
c) Cardiovascular disease
d) Proliferative retinopathy
e) Nephropathy
Question #30
QID: 6160
Topic: DM
Subject: Medicine
A 60-year-old black male is found to have type 2 diabetes mellitus. Which one of the
following should be ordered before initiating treatment with metformin
(Glucophage)?
a) Serum electrolytes
b) A serum creatinine level
c) A CBC
d) A lipid panel
e) A thyroid panel
Explanation:
Metformin is contraindicated in patients with renal dysfunction, because it is
associated with an increased incidence of lactic acidosis.
Question #31
QID: 127
Topic: Hyponatremia
Subject: Medicine
A 26-year-old man is admitted to the hospital for lethargy and confusion. The patient
has a 5 year history of severe hypertension that has been refractory to medical
therapy. Over the past few days, the man has become increasingly lethargic and
confused. On the day of admission, he was found in his home, lying on the floor and
non-responsive. His roommate called the ambulance, and the patient was brought to
the ED.
Laboratory values are as follows:
a) Aldosterone
b) Antidiuretic hormone (ADH)
c) Cortisol
d) Somatostatin
e) Thyroid stimulating hormone (TSH)
Explanation:
Hyponatremia is the most common electrolyte abnormality encountered in the
hospital. This patient is clearly hyponatremic. ADH works at the collecting ducts of
the nephron in the kidney to bring back water from the urine into the bloodstream.
This causes both the serum osmolality and serum sodium to drop as the serum
becomes more dilute.
Recall that aldosterone brings back sodium from the urine into the bloodstream and
kicks out serum potassium into the urine. Therefore, high levels of aldosterone would
present as hypokalemia with hypernatremia.
Question #32
QID: 678
Topic: Hyponatremia
Subject: Medicine
A 72-year-old man with a history of recently diagnosed small cell cancer of the lung,
presents to the emergency room following a witnessed, single, tonic-clonic seizure.
His serum sodium is 106 mmol/L.
Which one of the following is the best immediate approach to the management of his
electrolyte disturbance?
a) Fluid restriction
b) Intravenous normal saline at 25 mL/hr
c) Intravenous normal saline at 100 mL/hr
d) Intravenous 3% saline at 25 mL/hr
e) Intravenous 3% saline at 100 mL/hr
Severe hyponatremia (plasma Na < 109 mmol/L; effective osmolality < 238
mOsm/kg) in asymptomatic patients can be treated safely with stringent restriction of
water intake. However, treatment is more aggressive when neurologic symptoms (eg,
confusion, lethargy, seizures, coma) are present. The debate primarily concerns the
pace and degree of hyponatremia correction.
Many experts recommend that plasma Na be raised no faster than 1 mmol/L/h, but
replacement rates of up to 2 mmol/L/h for the first 2 to 3 h have been suggested for
patients with seizures. Regardless, the rise should be ≤ 10 mmol/L over the first 24 h.
More vigorous correction risks precipitation of osmotic demyelination syndrome.
Hypertonic (3%) saline (containing 513 mmol Na/L) may be used, but only with
frequent (q 2 to 4 h) electrolyte determinations. For patients with seizures or coma, ≤
100 mL/h may be administered over 4 to 6 h in amounts sufficient to raise the serum
Na 4 to 6 mmol/L.
> Fluid restriction (choice A) is incorrect. This patient's history doesn't reveal that she
was drinking large amount of fluids; while fluid restriction is normally the initial
treatment for syndrome of inappropriate antidiuretic hormone patients, this is
normally done in asymptomatic patients; once seizures develop, the most preferred
treatment is hypertonic saline.
> Intravenous normal saline (choice B) and (choice C) are incorrect. Treatment for
patients who present with seizure due to hyponatremia is done with hypertonic saline.
> Intravenous 3% saline 25ml/hr (choice D) would be insufficienct to correct the
patient's condition in a timely manner.
Question #33
QID: 6073
Topic: Hyponatremia
Subject: Medicine
Which one of the following is the treatment of choice for this patient?
a) Valsartan (Diovan)
b) Furosemide
c) Vasopressin (Pitressin)
d) Hypertonic saline
e) Conivaptan (Vaprisol)
Explanation:
This patient has severe hyponatremia manifested by confusion and seizures, a life-
threatening situation warranting urgent treatment with hypertonic (3%) saline. The
serum sodium level should be raised by only 1-2 mmol/L per hour, to prevent serious
neurologic complications. Saline should be used only until the seizures stop. Some
authorities recommend concomitant use of furosemide, especially in patients who are
likely to be volume overloaded, as this patient is, but it should not be used alone. The
arginine vasopressin antagonist conivaptan is approved for the treatment of euvolemic
or hypervolemic hyponatremia, but not in patients who are obtunded or in a coma, or
who are having seizures.
Question #34
QID: 157
Topic: Hypothyroidism
Subject: Medicine
a) Lithium level
b) TSH level
c) Reassurance
d) CBC
e) CMP
Explanation:
It is well established that lithium therapy can cause hypothyroidism in about 2-3 % of
patients. Once hypothyroidism is diagnosed, treatment of the condition should be
investigated rapidly with thyroxine.
A 38-year-old woman presents to you convinced that she has chronic fatigue
syndrome. The history reveals no evidence of fever or swollen glands, and no other
symptoms other than fatigue and difficulty sleeping. Her physical examination is
completely normal. She has a high thyroid-stimulating hormone level.
Explanation:
Hypothyroidism is underactivity of the thyroid gland that leads to inadequate
production of thyroid hormone and a slowing of vital body functions.
Insufficient thyroid hormone causes body functions to slow. Symptoms are subtle and
develop gradually. They may be mistaken for depression, especially among older
people. Facial expressions become dull, the voice is hoarse and speech is slow,
eyelids droop, and the eyes and face become puffy. Many people with hypothyroidism
gain weight, become constipated, and are unable to tolerate cold.
Diagnosis is made by obtaining TSH and FreeT4 levels. Treatment is with thyroid
replacement with synthroid.
Question #36
QID: 570
Topic: Hypothyroidism
Subject: Medicine
A young woman presents a feature of cold intolerance, fatigue, anorexia, weight gain
and dry skin. Her TSH is increased and freeT4 is low.
What is the most appropriate treatment?
a) Beta-blocker
b) Levothyroxine
c) Methimazole
d) Propylthiouracil
Explanation:
Hypothyroidism is thyroid hormone deficiency. It is diagnosed by clinical features
such as a typical facies, hoarse slow speech, and dry skin, and by low levels of thyroid
hormones. Management includes treatment of the underlying cause and administration
of thyroxine.
Question #37
QID: 761
Topic: Hypothyroidism
Subject: Medicine
An elderly woman notes the gradual onset of fatigue and feeling tired. She has gained
a significant amount of weight over the last year. She complains frequently that the
house is too cold and is turning up the thermostat. Constipation has become a
problem.
a) TSH is depressed
b) Anti-thyroglobulin antibody are elevated
c) Anti-thyroid microsomal antibodies are decreased
d) TRH is elevated
e) TSH is increased
Explanation:
Hypothyroidism is thyroid hormone deficiency. It is diagnosed by clinical features
such as a typical facies, hoarse slow speech, and dry skin, and by low levels of thyroid
hormones. Symptoms may include cold intolerance, constipation, forgetfulness,
fatigue, personality changes and weight gain.
Serum TSH is the most sensitive test. In primary hypothyroidism, there is no feedback
inhibition of the intact pituitary, and serum TSH is always elevated, whereas serum
free T4 is low. In secondary hypothyroidism, free T4 and serum TSH are low.
Question #38
QID: 902
Topic: Hypothyroidism
Subject: Medicine
Which one of the following is the most sensitive laboratory test that will be used to
diagnose this patient's condition?
Explanation:
Hypothyroidism is thyroid hormone deficiency. It is diagnosed by clinical features
such as a typical facies, hoarse slow speech, and dry skin, and by low levels of thyroid
hormones.
Serum TSH is the most sensitive test. In primary hypothyroidism, there is no feedback
inhibition of the intact pituitary, and serum TSH is always elevated, whereas serum
free T4 is low. In secondary hypothyroidism, free T4 and serum TSH are low
(sometimes TSH is normal but with decreased bioactivity). Management includes
treatment of the underlying cause and administration of thyroxine.
Question #39
QID: 980
Topic: Hypothyroidism
Subject: Medicine
A 28 year old woman presents to you with a three month history of fatigue, weight
gain, menstrual irregularity and declining memory. Which of the following laboratory
results is likely in the diagnosis of the underlying problem?
Explanation:
Hypothyroidism is common, especially among older people, particularly women; it
affects about 10% of older women. It can, however, occur at any age.
Insufficient thyroid hormone causes body functions to slow. Symptoms are subtle and
develop gradually. They may be mistaken for depression, especially among older
people. Facial expressions become dull, the voice is hoarse and speech is slow,
eyelids droop, and the eyes and face become puffy. Many people with hypothyroidism
gain weight, become constipated, and are unable to tolerate cold. The hair becomes
sparse, coarse, and dry, and the skin becomes coarse, dry, scaly, and thick.
Usually hypothyroidism can be diagnosed with one simple blood test: the
measurement of thyroid-stimulating hormone, which will be high due to a lack of
negative feedback. A second blood test is needed to measure the level of the thyroid
hormone T4 that is not bound by protein (free). A low level confirms the diagnosis of
hypothyroidism.
Question #40
QID: 1181
Topic: Hypothyroidism
Subject: Medicine
a) Granulocytopenia
b) Hypothyroidism
c) Recurrent laryngeal nerve damage
d) Thyroid carcinoma
Explanation:
Hypothyroidism is the main complication of radioactive iodine therapy, affecting up
to 70% of patients in 10 years. Radioactive iodine therapy is a safe and effective
treatment for Graves' disease because it can provide the same ablative effects of
surgery without the surgical complications. There is no evidence that this treatment
increases the risk for carcinoma.
Question #41
QID: 1250
Topic: Hypothyroidism
Subject: Medicine
a) Although she will have symptoms of hypothyroidism, she will not be given the
medication
b) Although she will not receive the medication, she will have no adverse effects
c) The medication will be administered daily while the nasogastric tube is clamped
d) The medication will be discontinued temporarily before the operation
e) She will receive the medication intravenously
Explanation:
Thyroxine is the hormone of choice for replacement therapy. It has a half-life of seven
days, and any alteration in dose is not reflected for four to six weeks. Therefore, it is
very unlikely that she will develop signs and symptoms of hypothyroidism. She will
not be given the medication either through the nasogastric tube or intravenously, nor
does she require a preoperative loading dose.
Question #42
QID: 5445
Topic: Hypothyroidism
Subject: Medicine
Question #43
QID: 5574
Topic: Hypothyroidism
Subject: Medicine
A 67-year-old female presents with progressive weakness, dry skin, lethargy, slow
speech, and eyelid edema. Of the following medications currently taken by this
patient, which one is most likely to be causing her symptoms?
a) Donepezil (Aricept)
b) Lithium
c) Lisinopril (Prinivil, Zestril)
d) Alendronate (Fosamax)
e) Glyburide (DiaBeta, Micronase)
Explanation:
This patient has classic signs of hypothyroidism. Of the drugs listed, only lithium is
associated with the development of hypothyroidism. In patients taking lithium, it is
recommended that in addition to regular serum lithium levels, thyroid function tests
including total free T , and TSH be obtained yearly.
Question #44
QID: 8788
Topic: Hypothyroidism
Subject: Medicine
An asymptomatic 55-year-old male visits a health fair, where he has a panel of blood
tests done. He brings the results to you because he is concerned about the TSH level
of 12.0 micro U/mL (N 0.45-4.5). His free T4 level is normal.
Which one of the following is most likely to be associated with this finding?
a) Atrial fibrillation
b) Reduced bone density
c) Systolic heart failure
d) Elevated LDL cholesterol
e) Type 2 diabetes mellitus
Explanation:
With subclinical thyroid dysfunction, TSH is either below or above the normal range,
free T4 or T3 levels are normal, and the patient has no symptoms of thyroid disease.
Subclinical hypothyroidism (TSH >10 U/mL) is likely to progress to overt
hypothyroidism. Patients with a serum thyroid-stimulating hormone level greater than
10 μU per mL have a higher incidence of elevated serum low-density lipoprotein
cholesterol concentrations.
Subclinical hyperthyroidism (TSH <0.1 U/mL) is associated with the development of
atrial fibrillation, decreased bone density, and cardiac dysfunction. Neither type of
subclinical thyroid dysfunction is associated with diabetes mellitus. There is
insufficient evidence of benefit to warrant early treatment of either condition.
Question #45
QID: 311
Topic: Addison's Disease
Subject: Medicine
A 35-year-old woman is brought to the ER in severe distress. She has had the "flu" for
three days. Past history reveals a six month history of fatigue and malaise. Physical
exam reveals BP 120/65 mm Hg supine, 90/58 mm Hg standing, and darkened areas
of skin on the knuckles, creases of the palm, elbows and an abdominal scar.
Laboratory values are Na 122 mmol/L, K 5.8 mmol/L, Cl 95 mmol/L, CO2 18
mmol/L.
Which one of the following laboratory tests is most likely to assist in confirming your
diagnosis?
Explanation:
Addison's disease (adrenal insufficiency) is an insidious, usually progressive
hypofunctioning of the adrenal cortex. It produces various symptoms, including
hypotension and hyperpigmentation, and can lead to adrenal crisis with cardiovascular
collapse.
Diagnosis is clinical and is made by finding elevated plasma ACTH with low plasma
cortisol. Treatment depends on the cause but generally includes hydrocortisone and
sometimes other hormones.
Question #46
QID: 2894
Topic: Addison's Disease
Subject: Medicine
A 40-year-old female presents with the gradual onset of weakness, fatigue, anorexia,
and nausea. She has lost 15 lb (7 kg) over the past 3 months.
Which one of the following is the best test to further evaluate this patient’s condition?
a) A TSH level
b) A cosyntropin (Cortrosyn) stimulation test
c) A plasma ACTH level
d) A dexamethasone suppression test
Explanation:
Primary adrenal insufficiency (Addison’s disease) is an uncommon condition in
clinical practice, and typically has an autoimmune etiology. Patients often experience
the insidious onset of weakness, fatigue, and gastrointestinal symptoms. Physical
findings typically include hypotension and diffuse hyperpigmentation of the patient’s
skin and mucous membranes. Due to the loss of aldosterone, serum sodium levels
tend to be low, and hyperkalemia is common.
In primary adrenal insufficiency, TSH and ACTH levels may be elevated, but this is
variable and ACTH continues to demonstrate diurnal variation. The dexamethasone
suppression test is used in the evaluation of cortisol excess.
Question #47
QID: 3301
Topic: Addison's Disease
Subject: Medicine
Which one of the following tests would most quickly determine if she has suffered
from acute adrenal cortex insufficiency (Addison’s disease)?
Explanation:
Destruction of the adrenal cortex by acute causes (infarction, hemorrhage, infection)
or chronic ones (granulomatous disease, immune destruction) results in a loss of
circulating levels of cortisol and aldosterone. The effects of this deficiency can
include electrolyte disturbances such as those seen in this patient, gastrointestinal
distress (including “acute abdomen,” diarrhea, nausea, etc.), orthostatic hypotension,
and patient fatigue. If the process is gradual, ACTH levels will increase, resulting in a
darkened skin tone. Measuring serum levels of ACTH may be helpful in determining
the cause of decreased cortisol levels, but is not a good first test of adrenal cortex
function. The dexamethasone suppresion test is used to evaluate glucocorticoid
excess.
The cosyntropin stimulation test does assess adrenal cortex function. This test
involves injecting cosyntropin, 250 µg intramuscularly or intravenously, and
measuring cortisol levels 60 minutes later. If the result shows an inadequate response
(cortisol level <495 nmol/L), adrenal cortical function is proven to be at least 90%
diminished.
Question #48
QID: 368
Topic: Pituitary Adenoma
Subject: Medicine
Explanation:
Enlargement of the pituitary gland is usually due to a tumor but may be due to
bleeding into the gland or involvement by some other disease, such as tuberculosis or
sarcoidosis. An enlarged pituitary gland may produce symptoms such as headaches.
Because the growing gland often presses on the optic nerve, which passes above the
pituitary gland, loss of vision may occur. Vision loss often initially affects only the
upper, outermost fields of vision in both eyes (bitemporal hemianopia).
Underproduction or overproduction of pituitary hormones may also occur. Treatment
depends on the cause of the enlargement.
> Craniopharyngioma occurs most commonly in children but also in men and women
in their 50s and 60s.
Question #49
QID: 2913
Topic: Pituitary Adenoma
Subject: Medicine
a) Graves’ disease
b) Thyroiditis
c) Struma ovarii
d) Thyrotropin-secreting pituitary adenoma
e) Factitious hyperthyroidism
The correct answer is d)
Explanation:
A normal or elevated serum thyrotropin level associated with an elevated free serum
thyroxine level is most consistent with a thyrotropin-secreting pituitary adenoma or a
thyroid hormone resistance syndrome. All of the other conditions listed are associated
with an elevated free thyroxine level, but would result in a suppressed serum
thyrotropin level.
Question #50
QID: 414
Topic: SIADH
Subject: Medicine
A 49-year-old man who smokes two packs of cigarettes a day presents with a lung
mass on x-ray and recent weight gain. Laboratory examination shows hyponatremia
with hyperosmolar urine.
a) Renal failure
b) Pituitary failure
c) Conn's syndrome
d) Cardiac failure
e) Inappropriate ADH (secretion)
Explanation:
SIADH (syndrome of inappropriate secretion of anti-diuretic hormone) is a common
paraneoplastic syndrome that affects the endocrine system. This syndrome is most
often associated with small-cell lung cancer; however, other cancers such as brain
tumors, leukemia, lymphoma, colon, prostate, and head and neck cancers can lead to
SIADH. SIADH is caused by the inappropriate production and secretion of arginine
vasopressin or antidiuretic hormone (ADH) by tumor cells.
Patients with SIADH may not have symptoms, especially in the early stages. When
symptoms do occur they are usually related to hyponatremia, which leads to central
nervous system toxicity if left untreated. Lab values will show concentrated urine in
the presence of plasma hypo-osmolality (hyponatremia).
Question #51
QID: 999
Topic: SIADH
Subject: Medicine
A 68 year old man presents to the emergency room with a four day history of
worsening headaches and confusion. He is on no medications. Physical examination
confirms the confusion, but is otherwise normal. His plasma sodium is found to be
117 mmol/L (normal 135 - 145 mmol/L) and plasma osmolality is 248 mmol/kg H20
(normal 285 - 295mmol/kg H20).
a) Hyperadrenalism
b) Syndrome of inappropriate antidiuretic hormone secretion
c) Congestive heart failure
d) Pseudohyponatremia
e) Hyperthyroidism
Explanation:
A cause of hyponatremia is the syndrome of inappropriate secretion of antidiuretic
hormone (SIADH). People who have SIADH have a low sodium level because the
pituitary gland secretes too much antidiuretic hormone. Hyponatremia also occurs in
people who have underactive adrenal glands and who thus excrete too much sodium
in the urine. This can result in a low plasma osmolality due to the excess water being
brought back into the bloodstream by the excess ADH.
When the sodium level in the blood falls quickly, symptoms tend to develop rapidly
and be more severe. The brain is particularly sensitive to changes in the sodium level
in the blood. Therefore, symptoms such as lethargy and confusion occur first. As
hyponatremia becomes more severe, muscle twitching and seizures may occur; stupor,
coma, and death may follow. The diagnosis of hyponatremia is made by measuring
the sodium level in the blood.
Question #52
QID: 3251
Topic: SIADH
Subject: Medicine
A 75-year-old white female presents with hyponatremia, with a serum level of 118
mEq/L, a urine osmolality 285 mOsm/kg H2O, and a serum osmolality of 242
mOsm/kg H2O. She complains of some fatigue, but is alert and oriented. Her blood
pressure is 136/82 mm Hg. She has normal thyroid, adrenal, cardiac, hepatic, and
renal function. You admit her to the hospital for treatment and observation.
Explanation:
This patient probably has the syndrome of inappropriate secretion of antidiuretic
hormone (SIADH). SIADH can be caused by CNS tumors, various infections such as
meningitis, and pneumonia. Several drugs can cause this condition, including
amiodarone, carbamazepine, SSRIs, and chlorpromazine. Laboratory results in
SIADH show hyponatremia <135 mmol/L, serum osmolality is usually <
280 mOsm/kg H2O, and is even generally lower than urine osmolality. However, it
should be noted that even if urine osmolality is lower than serum osmolality, it
should still be considered abnormally high if it is >100 mOsm/kg H2O in a
hyponatremic patient. Patients with hyponatremia should turn off ADH and have a
urine that is maximally dilute (ie, 50-100 mOsm/kg); however, in patients with
SIADH, the urinary osmolality is usually submaximally dilute (ie, >100 mOsm/kg).
Therefore, if you noticed a patient with hyponatreamia, serum osmolality 240
mOsm/kg H2O, and urine osmolality of 200 mOsm/kg H2O, that patient would still be
having labs consistent with SIADH even if the serum osmolality would be higher than
the urine osmolality.
In this fairly asymptomatic patient we have in the present case, initial management
should be free water restriction (choice C).
You see a patient with a serum sodium level of 122 mEq/L (N 135-145) and a serum
osmolality of 255 mOsm/kg H O (N 280-295). Which one of the following would best
correlate with a diagnosis of syndrome of inappropriate antidiuresis?
a) Fractional excretion of sodium <1%
b) Elevated urine osmolality
c) Elevated serum glucose
d) Elevated BUN
e) Low plasma arginine vasopressin
Explanation:
The syndrome of inappropriate antidiuresis (SIAD, formerly SIADH) is related to a
variety of pulmonary and central nervous system disorders in which hyponatremia
and hypo-osmolality are paradoxically associated with an inappropriately
concentrated urine. Most, but not all, cases are associated with increased levels of the
antidiuretic hormone arginine vasopressin (AVP). For a diagnosis of SIAD to be
made, the patient must be euvolemic and not on diuretics (within 24-48 hours), and
the urine osmolality must be high in conjunction with both low serum sodium and low
osmolality. The BUN is normal or low and the fractional excretion of sodium is >1%.
Fluid restriction (<800 cc/24 hours) over several days will correct the
hyponatremia/hypo-osmolality, but definitive treatment requires eliminating the
underlying cause, if possible. In the case of severe, acute hyponatremia with
symptoms (e.g., confusion, obtundation, seizures), hypertonic (3%) saline can be
slowly infused intravenously but can have dangerous neurologic side effects.
Elevated serum glucose levels give rise to a factitious hyponatremia, but not SIAD.
Question #54
QID: 435
Topic: Hyperparathyroidism
Subject: Medicine
Explanation:
Primary hyperparathyroidism is usually the result of a single benign adenoma. It is a
cause of secondary hypertension.
Question #55
QID: 481
Topic: Hyperparathyroidism
Subject: Medicine
A 53-year-old woman presents with lower back pain and increased thirst and
urination. She has nonspecific symptoms of tiredness, easy fatigability, sleep
disturbance and diminished ability to concentrate. Her past medical history is
significant for recurrent calcium oxalate renal stones. Initial labs show an increase in
serum calcium concentration.
Explanation:
Primary hyperparathyroidism is one of the most common causes of hypercalcemia
and should be considered in any individual with an elevated calcium level. A single
parathyroid adenoma is the underlying pathology in 85% of cases.
The symptoms have become known as "moans, groans, stones, and bones...with
psychic overtones". They include feelings of weakness and fatigue, depression, or
aches and pains. With more severe disease, a person may have a loss of appetite,
nausea, vomiting, constipation, confusion or impaired thinking and memory, and
increased thirst and urination. Patients may have thinning of the bones without
symptoms, but with risk of fractures.
Question #56
QID: 1597
Topic: Hyperparathyroidism
Subject: Medicine
A healthy 72 year old female comes to your office for a follow-up visit. She has
hypertension which is well controlled with an ACE inhibitor. Routine laboratory tests
are normal except for a serum calcium level of 10.8 mg/dL (N 8.5-10.5). A repeat
calcium level is 11.1 mg/dL.
Explanation:
In primary hyperparathyroidism, hypercalcemia is the result of excessive PTH
secretion by one or more abnormal, enlarged parathyroid glands. Laboratory findings
in most patients with primary hyperparathyroidism reflect the mild clinical
presentation of the disorder. The serum calcium level is often 1 mg/dL or more above
the upper limits of normal.
>Bone radiographs may show the classic changes of subperiosteal bone resorption in
the occasional patient with hyperparathyroidism, but in most cases they are normal or
may show osteopenia.
>Osteocalcin is an osteoblast-specific protein. It is a marker of increased skeletal
turnover, and it is usually not indicated clinically. The development of highly
sensitive and specific assays for intact, largely active PTH has simplified the
assessment of parathyroid activity.
>Bone densitometry is a test to determine the degree of osteoporosis.
>Hypercalcemia is not a side effect of ACE inhibitor use
Question #57
QID: 2748
Topic: Hyperparathyroidism
Subject: Medicine
Routine blood tests frequently reveal elevated calcium levels. When this evaluation is
associated with elevated parathyroid hormone levels, which one of the following is an
indication for parathyroid surgery?
a) Age > 50
b) Kidney stones
c) Serum calcium 0.5 mg/dL above the upper limit of normal
d) Concurrent hyperthyroidism
e) Increased bone density
Explanation:
Indications for parathyroid surgery include kidney stones, age less than 50, a serum
calcium level greater than 1 mg/dL above the upper limit of normal, and reduced bone
density. Hyperthyroidism is not a factor in deciding to perform parathyroid surgery.
Question #58
QID: 2960
Topic: Hyperparathyroidism
Subject: Medicine
a) Lithium therapy
b) Parathyroid cancer
c) Multiple endocrine neoplasia
d) An adenoma of a single parathyroid gland
Explanation:
An adenoma of a single parathyroid gland is the cause of primary
hyperparathyroidism in 85% of cases. Other fairly frequent causes are multiple
adenomas and hypertrophy of all four glands. Cancer and multiple endocrine
neoplasias are rare. Lithium therapy may cause hyperparathyroidism, but this is not
considered primary hyperparathyroidism.
Question #59
QID: 5500
Topic: Hyperparathyroidism
Subject: Medicine
A 59-year-old male reports nausea, vomiting, and progressive fatigue for the past few
months. At his last visit, 6 months ago, his blood pressure was poorly controlled and
hydrochlorothiazide was added to his Beta-blocker therapy. At this visit he appears
moderately dehydrated on examination. Laboratory testing reveals a serum calcium
level of 3.19 mmol/L, a BUN level of 12.9 mmol/L and a creatinine level of 220
µmol/L. A CBC, albumin level, and electrolyte levels are normal. His intact
parathyroid hormone level is reported a few days later, and is 60 pg/mL (N 10-65).
What is the most likely cause of his hypercalcemia?
a) Renal failure
b) Hyperparathyroidism
c) Milk alkali syndrome
d) Sarcoidosis
Explanation:
Many patients have mild hyperparathyroidism that becomes evident only with an
added calcium load. Thiazide diuretics reduce calcium excretion and can cause overt
symptoms in a patient whose hyperparathyroidism would otherwise have remained
asymptomatic. The finding of a normal parathyroid hormone (PTH) level in a patient
with hypercalcemia is diagnostic for hyperparathyroidism, since PTH should be
suppressed in the presence of elevated calcium. Symptomatic hypercalcemia causes
dehydration because of both intestinal symptoms and diuresis. Reversible renal
insufficiency can result, and can become permanent if it is long-standing and severe.
Conversely, renal failure usually causes hypocalcemia, but can cause hypercalcemia
resulting from tertiary hyperparathyroidism. This develops after severe
hyperphosphatemia and vitamin D deficiency eventually produce hypersecretion of
PTH. This patient’s renal insufficiency is not severe enough to cause tertiary
hyperparathyroidism. Milk alkali syndrome is hypercalcemia resulting from a chronic
overdose of calcium carbonate, and is becoming more common as more patients take
calcium and vitamin D supplements. In milk alkali syndrome, and other causes of
hypercalcemia such as sarcoidosis, the PTH level is appropriately suppressed.
Question #60
QID: 5632
Topic: Hyperparathyroidism
Subject: Medicine
A 35-year-old male with a previous history of kidney stones presents with symptoms
consistent with a recurrence of this problem. The initial workup reveals elevated
serum calcium. Which one of the following tests would be most appropriate at this
point?
a) Serum calcitonin
b) 24-hour urine for calcium and phosphate
c) Serum phosphate and magnesium
d) Serum parathyroid hormone
e) Spot urine for microalbumin
Explanation:
A patient with a recurrent kidney stone and an elevated serum calcium level most
likely has hyperparathyroidism, and a parathyroid hormone (PTH) level would be
appropriate. Elevated PTH is caused by a single parathyroid adenoma in
approximately 80% of cases. The resultant hypercalcemia is often discovered in
asymptomatic persons having laboratory work for other reasons. An elevated PTH by
immunoassay confirms the diagnosis. In the past, tests based on renal responses to
elevated PTH were used to make the diagnosis. These included blood phosphate,
chloride, and magnesium, as well as urinary or nephrogenous cyclic adenosine
monophosphate. These tests are not specific for this problem, however, and are
therefore not cost-effective. Serum calcitonin levels have no practical clinical use.
Question #61
QID: 456
Topic: Diabetes Insipidus
Subject: Medicine
A 48-year-old woman presents with thirst and polyuria in the order of 4-5 L per 24
hours. The urinalysis (urine microscopy) reveals no glucose or albumin and a normal
sediment. The urine specific gravity is 1.001, and the serum sodium 137 mmol/L.
After 8 hours of no fluid intake, the urine specific gravity is 1.001 and the serum
sodium 149 mmol/L.
a) Primary polydipsia
b) Chronic renal failure
c) Diabetes insipidus
d) Syndrome of inappropriate antidiuretic hormone secretion
e) Osmotic diuresis
Question #62
QID: 581
Topic: Diabetes Insipidus
Subject: Medicine
Explanation:
This patient likely has developed damage to the pituitary gland which has affected the
release of anti-diuretic hormone (ADH).
Question #63
QID: 6238
Topic: Diabetes Insipidus
Subject: Medicine
Explanation:
This patient has diabetes insipidus, which is caused by a deficiency in the secretion or
renal action of arginine vasopressin (AVP). AVP, also known as antidiuretic
hormone, is produced in the posterior pituitary gland and the route of secretion is
generally regulated by the osmolality of body fluid stores, including intravascular
volume. Its chief action is the concentration of urine in the distal tubules of the
kidney. Both low secretion of AVP from the pituitary and reduced antidiuretic action
on the kidney can be primary or secondary, and the causes are numerous. Patients
with diabetes insipidus present with profound urinary volume, frequency of urination,
and thirst. The urine is very dilute, with osmolality <300 mOsm/L. Further workup
will help determine the specific type of diabetes insipidus and its cause, which is
necessary in order to implement appropriate treatment. Low levels of aldosterone,
plasma renin activity, or angiotensin would cause abnormal blood pressure,
electrolyte levels, and/or renal function. Insulin deficiency results in diabetes mellitus.
Question #64
QID: 467
Topic: Hypercalcemia
Subject: Medicine
An elderly female is on Tamoxifen therapy for advanced breast cancer with bone
metastases. She develops increased thirst, increased urination, disorientation, nausea,
vomiting, confusion and agitation.
Explanation:
Hypercalcemia is the most common paraneoplastic syndrome associated with cancer.
Symptoms and signs of hypercalcemia may be noted when total serum Ca is > 12
mg/dL (> 3 mmol/L). These signs can include anorexia, GI reflux, vomiting, lethargy
or seizures or generalized irritability, and hypertension. Other symptoms and signs
include constipation, dehydration, feeding intolerance, and failure to thrive.
Question #65
QID: 816
Topic: Hypercalcemia
Subject: Medicine
Which one of the following treatment strategies represents the best initial
management of her hypercalcemia?
Explanation:
Hypercalcemia is total serum Ca > 12 mg/dL (3 mmol/L), or ionized Ca > 6 mg/dL
(1.5 mmol/L). The most common cause is iatrogenic. GI signs may occur (anorexia,
vomiting, constipation) and sometimes lethargy or seizures.
Treatment: give IV fluids. Then, once the patient is well hydrated, give furosemide
(i.e. a loop diuretic) to cause calcium diuresis. Thiazides are contraindicated because
they increase serum calcium levels. Other treatments include phosphorus
administration (use oral phosphorus; intravenous administration can be dangerous),
calcitonin, diphosphonates, plicamycin, or prednisone. Correction of the underlying
cause of hypercalcemia is the ultimate goal.
Question #66
QID: 1606
Topic: Hypercalcemia
Subject: Medicine
A 65 year old male with known metastatic lung cancer is hospitalized because of
decreased appetite, lethargy, and confusion of 2 weeks’ duration.
Explanation:
Initial management of hypercalcemia of malignancy calls for fluid replacement with
normal saline to correct the volume depletion that is invariably present and to enhance
renal calcium excretion. The use of loop diuretics such as furosemide should be
restricted to patients in danger of fluid overload, since these drugs can aggravate
volume depletion and are not very effective alone in prompting renal calcium
excretion. Although intravenous pamidronate has become the mainstay for the
hypercalcemia of malignancy, it is considered only after the hypercalcemic patient has
been rendered euvolemic by saline repletion. The same is true for the other calcium-
lowering agents listed.
Question #67
QID: 1721
Topic: Hypercalcemia
Subject: Medicine
a) Furosemide (Lasix)
b) Verapamil (Calan, Isoptin)
c) Enalapril (Vasotec)
d) Hydrochlorothiazide
e) Allupurinol (Zyloprim)
The correct answer is d)
Explanation:
While thiazide diuretics do not cause hypercalcemia by themselves, they can
exacerbate the hypercalcemia associated with primary hyperparathyroidism. Thiazides
decrease the renal clearance of calcium by increasing distal tubular calcium
reabsorbtion. Furosemide tends to lower serum calcium levels and is used in the
treatment of hypercalcemia. None of the other medications would be expected to
significantly affect the serum calcium level in this patient.
Question #68
QID: 2907
Topic: Hypercalcemia
Subject: Medicine
Which one of the following is the most likely cause of hypercalcemia in a patient with
hypercalcemia and hyperphosphatemia?
a) Vitamin D excess
b) Primary hyperparathyroidism
c) Humoral hypercalcemia of malignancy
d) Exogenous calcitonin administration
Explanation:
The action of parathyroid hormone (PTH) causes phosphate loss through the kidneys;
consequently, hypercalcemia due to PTH excess is usually associated with
hypophosphatemia. Humoral hypercalcemia of malignancy is associated either with
osteolysis, in which case phosphate levels are unaffected, or with elevated levels of a
PTH-related peptide, which would also be associated with hypophosphatemia.
Calcitonin is a weak inhibitor of PTH action and is not associated with hypercalcemia.
Vitamin D increases the gastrointestinal absorption of both calcium and phosphorus;
excess levels are therefore associated with elevated levels of both calcium and
phosphorus. This helps to distinguish vitamin D excess from other causes of
hypercalcemia.
Question #69
QID: 3126
Topic: Hypercalcemia
Subject: Medicine
a) fluid restriction
b) bisphosphonates intravenously
c) phosphate depletion
d) aggressive rehydration
e) diuresis with furosemide (Lasix)
Explanation:
Hypercalcemia of malignancy should be treated initially with aggressive rehydration,
followed by diuresis with furosemide, phosphorus replacement if hypophosphatemia
is present, and intravenous bisphosphonates. Adjunctive therapies include dialysis,
glucocorticoids, calcitonin, plicamycin, and gallium nitrate.
Question #70
QID: 5491
Topic: Hypercalcemia
Subject: Medicine
A 49-year-old female who takes multiple medications has a chemistry profile as part
of her routine monitoring. She is found to have an elevated calcium level. All other
values on the profile are normal, and the patient is not currently symptomatic. Follow-
up testing reveals a serum calcium level of 2.8 mmol/ and a parathyroid hormone
level of 80 pg/mL (N 10-65). Which one of the following should be discontinued for 3
months before repeat laboratory evaluation and treatment?
a) Lithium
b) Furosemide (Lasix)
c) Raloxifene (Evista)
d) Calcium carbonate
e) Vitamin D
Explanation:
Lithium therapy can elevate calcium levels by elevating parathyroid hormone
secretion. This duplicates the laboratory findings seen with mild primary
hyperparathyroidism. If possible, lithium should be discontinued for 3 months before
reevaluation (SOR C). This is most important for avoiding unnecessary parathyroid
surgery.
Question #71
QID: 6047
Topic: Hypercalcemia
Subject: Medicine
Which one of the following is most likely to reveal the cause of this patient’s elevated
calcium?
a) A 24-hour urine calcium level
b) A repeat serum calcium level in 4-6 weeks
c) A serum 25-hydroxyvitamin D level
d) A serum calcitonin level
e) A serum intact parathyroid hormone level
Explanation:
Primary hyperparathyroidism and malignancy are the most common causes of
hypercalcemia, accounting for about 90% of cases. An intact parathyroid hormone
(PTH) level should be obtained initially, as the results will indicate what kind of
additional evaluation is needed. Vitamin D and urine calcium studies are useful in
evaluating hypercalcemia, but a PTH level should be obtained first. It would not be
appropriate to wait for a repeat calcium level in 4-6 weeks, because this patient has
nephrolithiasis and a calcium level 12 mg/dL, indicating a possible need for surgery
or perhaps a malignancy. Calcitonin levels generally are not necessary in the
evaluation of hypercalcemia.
Question #72
QID: 6062
Topic: Hypercalcemia
Subject: Medicine
Which one of the following is the best initial management for hypercalcemic crisis?
a) Intravenous furosemide
b) Intravenous pamidronate (Aredia)
c) Intravenous plicamycin (Mithramycin)
d) Intravenous saline
Explanation:
The initial management of hypercalcemic crisis involves volume repletion and
hydration. The combination of inadequate fluid intake and the inability of
hypercalcemic patients to conserve free water can lead to calcium levels over 14-15
mg/dL. Because patients often have a fluid deficiency of 4-5 liters, delivering 1000
mL of normal saline during the first hour, followed by 250-300 mL/hour, may
decrease the hypercalcemia to less than critical levels (<13 mg/dL). If the clinical
status is not satisfactory after hydration alone, then renal excretion of calcium can be
enhanced by saline diuresis using furosemide. Intravenous pamidronate, a
diphosphonate, reduces the hypercalcemia of malignancy and is best used in the semi-
acute setting, since calcium levels do not start to fall for 24 hours. The same is true for
intravenous plicamycin.
Question #73
QID: 11195
Topic: Hypercalcemia
Subject: Medicine
a) Bisphosphonates
b) Calcitonin
c) Saline infusion
d) Haemodialysis
e) Glucocorticoids
Explanation:
Saline infusion (choice C) is the first line of management for severe hypercalcaemia.
Hypercalcaemia causes dehydration due to vomiting and/or renal insufficiency.
Hydration on its own can lower calcium levels significantly and alleviates the
stuporous state.
Question #74
QID: 480
Topic: Anion Gap
Subject: Medicine
A patient has the following values of Na 140 mmol/L, K 4.0 mmol/L, Cl 100 mmol/L,
HCO3 24 mmol/L. Calculate the anion gap.
a) 122
b) 40
c) 16
d) 18
e) 64
Explanation:
Anion gap is calculated by the sum of cations minus anions as follows: Anion gap =
(Na+K)-(CL+HCO3).
However, the potassium is frequently ignored because potassium concentrations,
being very low, usually have little effect on the calculated gap. This leaves the
following equation:
= ( [Na+] ) − ( [Cl−]+[HCO3−] )
Question #75
QID: 10689
Topic: Anion Gap
Subject: Medicine
An inpatient with advanced liver disease started deteriorating over the last two days.
You were called to see him. His heart rate was 97 per minute, respiratory rate 27 per
minute and blood pressure 100/70 mmHg. You ordered some investigations. You are
presented with the following results:
Explanation:
The question here is not about calculation of plasma anion gap but about the baseline
plasma anion gap (expected normal plasma anion gap). In a patient with normal
plasma albumin, the baseline anion gap is 12 (range 10 - 14). Baseline anion gap
should be decreased by 3 for each 10 g/L fall in serum albumin (from normal average
of 40 g/L). Thus, in this patient baseline anion gap is 6 (12 - 6).
Or use: (0.25 x (44 - albumin)
Correction of baseline plasma anion gap for plasma albumin: baseline anion gap
should be decreased by 3 for each 10 g/L fall in serum albumin.
Question #76
QID: 10706
Topic: Anion Gap
Subject: Medicine
Blood pH 7.32
PaCO 2 30 mm Hg
Serum bicarbonate 15 mmol/L
Serum sodium 143 mmol/L
Serum chloride 110 mmol/L
Which of the following is least likely to be the cause of metabolic acidosis in this
patient?
a) Starvation
b) Alcohol consumption
c) Methanol poisoning
d) Salicylate poisoning
e) Diarrhea
Explanation:
The plasma anion gap of this patient is 18 {143 - (110 + 15)} which is higher than the
normal value of 12 (range 10 to 14). Diarrhea causes metabolic acidosis with normal
anion gap and is unlikely to be the cause of metabolic acidosis in this patient.
Question #77
QID: 484
Topic: Vitamins
Subject: Medicine
a) Vitamin A
b) Vitamin B6
c) Vitamin C
d) Vitamin D
Explanation:
Vitamins are substances that our body needs to grow and develop normally. There are
13 vitamins we need. They are vitamins A, C, D, E, K and the B vitamins (thiamine,
riboflavin, niacin, pantothenic acid, biotin, vitamin B-6, vitamin B-12 and folate).
You can usually get all your vitamins from the foods you eat. Your body can also
make vitamins D and K. People who eat a vegetarian diet may need to take a vitamin
B12 supplement.
Question #78
QID: 1582
Topic: Vitamins
Subject: Medicine
Which one of the following clinical presentations is most consistent with vitamin D
deficiency in the aged?
Explanation:
Vitamin D deficiency is being recognized more frequently among the elderly,
especially in the nursing home or other settings where inactivity, nutritional
deficiency, and lack of sunlight exposure combine to increase the propensity for
deficiency. Vitamin D deficiency has been associated with abnormalities in bone
metabolism, primarily osteomalacia. Clinically this can present as muscle weakness,
limb pain, and impaired physical function. Bleeding gums and fatigue may be
presenting symptoms of scurvy (vitamin C deficiency). High-output cardiac failure
with vasodilation, dermatitis, and neuropathy is the clinical presentation of thiamine
deficiency. The combination of diarrhea, scaly dermatitis, and dementia is one of the
clinical presentations of niacin deficiency.
Question #79
QID: 8733
Topic: Vitamins
Subject: Medicine
The daily intake of vitamins and minerals recommended varies according to sex, age,
and condition. The recommended daily allowance of vitamin D is greatest for which
one of the following?
Question #80
QID: 508
Topic: Hypoglycemia
Subject: Medicine
A patient with known type II diabetes mellitus who is treated with Clorpropamide,
presents to ED unconscious. His blood sugar level is very low. You administer 1
ampule of D50 and the patient wakes up promptly.
Explanation:
This patient should be admitted. Hypoglycemia causes sweating, nausea, warmth,
anxiety, tremulousness, palpitations, and possibly hunger and paresthesias.
Insufficient glucose supply to the brain causes headache, blurred or double vision,
confusion, difficulty speaking, seizures, and coma. In controlled settings, autonomic
symptoms begin at or beneath a plasma glucose level of about 60 mg/dL (3.33
mmol/L), whereas CNS symptoms occur at or below a glucose level of about 50
mg/dL (2.78 mmol/L).
Question #81
QID: 681
Topic: Hypoglycemia
Subject: Medicine
Which one of the following would result in these clinical and laboratory findings?
a) Insulinoma
b) Reactive hypoglycemia from eating
c) Glucagonoma
d) Insulin taken surreptitiously
Explanation:
C-peptide is measured to differentiate insulin produced by the body from insulin
injected into the body. When insulin is synthesized by the beta cells of the pancreas, it
is produced as a large molecule (a propeptide). This molecule is then split into two
pieces: insulin and C-peptide. The function of C-peptide is not known.
The C-peptide level may be measured in a patient with type 1 diabetes to see if any
insulin is still being produced by the body. It may also be measured in the evaluation
of hypoglycemia to see if the person's body is producing too much insulin.
Normal values in a patient requiring insulin injections indicate that the person's body
is still producing some insulin. Normal values in a patient who has low blood sugar
indicate that the patient is making too much insulin.
a) Cushing's triad
b) Whipple's triad
c) Samter's triad
d) Virchow's triad
e) Beck's triad
Explanation:
Hypoglycemia unrelated to exogenous insulin therapy is an uncommon clinical
syndrome characterized by low plasma glucose level, symptomatic sympathetic
nervous system stimulation, and CNS dysfunction. Many drugs and disorders cause it.
Question #83
QID: 6214
Topic: Hypoglycemia
Subject: Medicine
A 35-year-old white male who has had diabetes mellitus for 20 years begins having
episodes of hypoglycemia. He was previously stable and well controlled and has not
recently changed his diet or insulin regimen.
Which one of the following is the most likely cause of the hypoglycemia?
a) Spontaneous improvement of beta-cell function
b) Renal disease
c) Reduced physical activity
d) Insulin antibodies
Explanation:
The most common cause of hypoglycemia in a previously stable, well-controlled
diabetic patient who has not changed his or her diet or insulin dosage is diabetic renal
disease. Exogenous insulin is normally metabolized by the kidney. However, when
there is impairment of kidney function, the half-life of insulin is prolonged because of
lower levels of degradation. Therefore, in patients with diabetes (treated with insulin)
and moderate to severe kidney dysfunction, the frequency of hypoglycemic episodes
may be frequent.
Question #84
QID: 610
Topic: Hyperaldosteronism
Subject: Medicine
What is the most sensitive means of differentiating primary from secondary causes of
hyperaldosteronism?
a) Saline infusion test
b) Oral salt loading test
c) Captopril test
d) Aldosterone-to-renin ratio
e) Postural testing
Explanation:
The aldosterone-to-renin ratio (ARR) - that is, the ratio of plasma aldosterone to
plasma renin activity (PRA) - is the most sensitive means of differentiating primary
from secondary causes of hyperaldosteronism. It can be obtained under random
conditions of sodium intake. The principle behind this test is that as aldosterone
secretion rises, PRA (which measures the rate of production of angiotensin I from
endogenous angiotensinogen) in ex vivo testing should fall because of sodium
retention. This negative feedback response should occur when the aldosterone levels
are supraphysiologic for that individual patient, and PRA may fall well before plasma
aldosterone is clearly increased. Patients should be normokalemic because
hypokalemia suppresses aldosterone secretion.
> The saline infusion test (choice A) can confirm autonomous aldosterone secretion.
> The oral salt loading test (choice B) consists of administration of sodium chloride
tablets with an ad libitum diet for 3 days, followed by a 24-hour urinary aldosterone
measurement. (increased urinary aldosterone and sodium excretion confirms primary
hyperaldosteronism)
> The captopril test (choice C) has also been used for screening. Its use is based on
the principle that inhibition of angiotensin II production should not affect autonomous
secretion of aldosterone in primary aldosteronism.
> Posural test (choice E) is used for differentiating aldosterone-producing adenoma
from other primary hyperaldosteronism.
Question #85
QID: 744
Topic: Hyperaldosteronism
Subject: Medicine
You are asked to see a 48-year-old male patient because of persistent hypertension
despite taking metoprolol, enalapril and nifedipine. There is no history of palpitations
or flushing, and the patient does not use any nonsteroidal anti-inflammatory drugs.
Blood pressure in your office was found to be elevated at 160/110 mmHg. Recent
laboratory testing shows sodium: 146 mmol/L; potassium: 2.4 mmol/L and creatinine:
85 µmol/L. There is no proteinuria on urinalysis.
Which one of the following is the most appropriate test to arrange at this time?
Explanation:
Aldosterone, a hormone produced and secreted by the adrenal glands, signals the
kidneys to excrete less sodium and more potassium.
Hyperaldosteronism can be caused by a tumor (usually a noncancerous adenoma) in
the adrenal gland (a condition called Conn's syndrome), although sometimes both
glands are involved and are overactive. Sometimes hyperaldosteronism is a response
to certain diseases, such as very high blood pressure (hypertension) or narrowing of
one of the arteries to the kidneys.
The amount of salt in the diet and medications, such as over-the-counter pain relievers
of the non-steroid class (such as Motrin and Advil), diuretics (water pills), beta
blockers, steroids, angiotensin-converting enzyme (ACE) inhibitors, and oral
contraceptives can affect the test results. You should ask your patient to stop taking
some of his medicines for 2 weeks before the test.
Screening test:
Random plasma aldosterone/plasma renin activity (PRA) ratio - Because this ratio is
fairly constant over many physiologic conditions, it can be used as a screening test.
Normal values are less than 270 when aldosterone concentration is expressed in
pmol/L, or are less than 10 when aldosterone concentration is expressed in ng/dL.
Confirmatory test:
The 24-hour urinary aldosterone (U-Aldo) excretion test is one of the most useful
confirmatory diagnostic tools.
Question #86
QID: 1032
Topic: Hyperaldosteronism
Subject: Medicine
Explanation:
In primary hyperaldosteronism (Conn’s disease), overproduction of aldosterone leads
to fluid retention and increased blood pressure, weakness, and, rarely, periods of
paralysis. Hyperaldosteronism can be caused by a tumor in the adrenal gland or may
be a response to some diseases. High aldosterone levels can cause high blood pressure
and low potassium levels; low potassium levels may cause weakness, tingling, muscle
spasms, and periods of temporary paralysis.
If hyperaldosteronism is suspected, both levels of sodium and potassium should be
ordered. Aldosterone levels may also be measured. If they are high, spironolactone or
eplerenone, drugs that block the action of aldosterone, may be given to see if the
levels of sodium and potassium return to normal. In Conn's syndrome, the levels of
renin are also very low.
> Urine metanephrines testing is primarily used to help detect and rule out
pheochromocytomas in symptomatic patients.
> When too much aldosterone is being produced, adrenal glands should be examined
for a noncancerous tumor (adenoma). computed tomography (CT) or magnetic
resonance imaging (MRI) can be helpful.
> Glucose level is normal, glucose tolerance test is not indicated.
Question #87
QID: 1034
Topic: Hyperaldosteronism
Subject: Medicine
A 29 year old has her blood pressure at 180/105 mmHg. She admits to stress at work
and states that her father was diagnosed as hypertensive at the age of 60 but, thus far,
has not required treatment.
Na 146 mmol/L
K 2.8 mmol/L
Urea 4.2 mmol/L
Creatinine 92 mmol/L
Which of the following secondary causes of hypertension does this patient most likely
have?
Explanation:
In hyperaldosteronism, overproduction of aldosterone leads to fluid retention and
increased blood pressure. When too much aldosterone is being produced, doctors
examine the adrenal glands for a noncancerous tumor (adenoma).
Question #88
QID: 1940
Topic: Hyperaldosteronism
Subject: Medicine
Explanation:
Secondary hyperaldosteronism is characterized by sodium retention, and thus
decreased urinary sodium excretion, while potassium secretion is normal to increased.
Question #89
QID: 648
Topic: Hyperkalemia
Subject: Medicine
A patient admitted to the hospital has a basic metabolic panel (BMP) drawn and sent
to the lab. The serum sodium, calcium, and bicarbonate are all normal except the
serum potassium was 6.4 mmol/L. The patient is asymptomatic.
What is the most appropriate next step?
a) Dextrose
b) Repeat the BMP + ECG
c) Calcium gluconate
d) Insulin
e) BUN and Creatinine levels
The correct answer is b)
Explanation:
High levels of potassium in an asymptomatic patient may be due to lab error. To
gather enough information for diagnosis, the measurement of potassium needs to be
repeated, as the elevation can be due to hemolysis in the first sample.
Hyperkalemia is defined as a potassium level greater than 5.5 mmol/L. Ranges are as
follows:
5.5-6.0 mmol/L - Mild
6.1-7.0 mmol/L - Moderate
7.0 mmol/L and greater - Severe
Question #90
QID: 2892
Topic: Hyperkalemia
Subject: Medicine
Which one of the following drugs would decrease his risk of hyperkalemia if used
concomitantly with an ACE inhibitor?
a) Furosemide (Lasix)
b) Naproxen (Naprosyn)
c) Celecoxib (Celebrex)
d) Spironolactone (Aldactone)
e) Propranolol (Inderal)
Explanation:
The geriatric population is at increased risk of hyperkalemia associated with the use
of ACE inhibitors, especially in the presence of renal disease. Drugs that increased
this risk, if used concomitantly with an ACE inhibitor, including β-blockers, NSAIDs
(including selective COX-2 inhibitors), and potassium-sparing diuretics. Other
diuretics, such as furosemide, enhance excretion of potassium by increasing delivery
of sodium to the collecting duct.
Question #91
QID: 2941
Topic: Hyperkalemia
Subject: Medicine
A previously healthy 42-year-old male who is not on any medications has blood
drawn for a routine preemployment evaluation. Laboratory studies are normal except
for a potassium level of 5.4 mmol/L (N 3.5-5.0).
Which one of the following would be the most appropriate next step?
Explanation:
Given the absence of underlying renal problems or medication use, the most likely
cause of this patient’s elevated potassium level is pseudohyperkalemia.
Pseudohyperkalemia occurs when the laboratory value does not accurately reflect
actual serum values. The most common cause is lysis of red blood cells during
phlebotomy collection. It can be excluded by repeating the blood sample collection as
nontraumatically as possible.
Potassium values less than 6mEq/L are not usually clinically significant and do not
cause EKG changes. Treatment with sodium polystyrene sulfonate, intravenous
calcium, intravenous insulin, and/or an inhaled β2-agonist are necessary only if
pseudohyperkalemia has been ruled out and/or there are clinical indicators such as
EKG changes, a rapid rise of serum potassium, decreased renal function, and/or
significant acidosis.
Question #92
QID: 3318
Topic: Hyperkalemia
Subject: Medicine
a) Order another serum potassium level but make sure the patient repeatedly clenches
his
fist during the phlebotomy
b) Order another metabolic profile but make sure the sample stands for several hours
before being centrifuged
c) Reassure the patient that the potassium elevation is likely due to a laboratory error
and
disregard the result
d) Temporarily discontinue the lisinopril and order a repeat potassium level
Explanation:
Pseudo-hyperkalemia is most commonly caused by lysis of red cells released from
platelets in clotted serum. Clenching the fist repeatedly can aggravate red cell
damage. As the patient is on an ACE inhibitor that can raise potassium levels,
reassurance without investigation would not be appropriate. A plasma potassium level
does not reflect potassium released from cells damaged by phlebotomy.
Question #93
QID: 5488
Topic: Hyperkalemia
Subject: Medicine
A 75-year-old female is admitted to the hospital with a change in mental status. The
initial workup includes a chemistry profile that reveals a plasma potassium level of
6.4 mEq/L (N 3.7-5.2). Which one of the following should be given now to rapidly
lower the plasma potassium level?
a) Corticosteroids
b) Albuterol
c) Sodium polystyrene sulfonate
d) 0.45% saline
e) Acute hemodialysis
Explanation:
Severe hyperkalemia (>6.0 mEq/L) requires aggressive treatment. Calcium gluconate
has no effect on the plasma potassium level, but it should be given first, as it rapidly
stabilizes the membranes of cardiac myocytes, reducing the risk of cardiac
dysrhythmias.
Therapies that translocate potassium from the serum to the intracellular space should
be instituted next, as they can quickly (albeit temporarily) lower the plasma
concentration of potassium. These interventions include sodium bicarbonate, glucose
with insulin, and albuterol.
Total body potassium can be lowered with sodium polystyrene sulfonate, but this
takes longer to affect the plasma potassium level than translocation methods. In the
most severe cases, acute hemodialysis can be instituted.
Question #94
QID: 11405
Topic: Hyperkalemia
Subject: Medicine
A known diabetic patient presents to emergency department with oliguria and high
blood pressure. With appropriate work-up, the diagnosis of acute renal failure is
established. His plasma glucose and serum potassium are 17 mmol/L and 7.2 mmol/L,
respectively.
Explanation:
Insulin (choice C) is expected to lower blood potassium and is indicated in the
treatment of patients with severe hyperkalemia (serum potassium > 7.0 mmol/L).
Insulin stimulates Na+/H+ /antiporter increasing intracellular sodium. This increase in
intracellular sodium stimulates the Na+/K+ ATPase, which would pump potassium
into the cells lowering plasma potassium. A drop of plasma potassium up to 1.5
mmol/L can be achieved with administration of 10 to 20 units of regular insulin and
the effect can last for several hours. In this patient, glucose administration is not
required because blood glucose is already high.
> Calcium whether given orally (choice A) or intravenously (choice B) does not lower
plasma potassium. However, IV calcium is the first line treatment of severe
hyperkalemia. Calcium does not lower plasma potassium but counteracts the effects
of hyperkalemia. Specifically, high extracellular calcium decreases excitability of
nerve and muscles cells by raising threshold potential, the membrane potential at
which an all or none action potential develops.
> Angiotensin converting enzyme inhibitor (ACEI) (choice D) may worsen
hyperkalemia by inducing secondary hypoaldosteronism. Aldosterone stimulates
secretion of potassium in exchange for sodium in the collecting ducts or the renal
tubules. Indeed if this patient is on an ACEI, this drug should be discontinued not
only because it might worsen hyperkalemia but also because it interrupts
glomerulotubular balance and might compromise recovery of renal function.
> Intravenous propranolol (choice E), beta-2-adrenergic blocker, may also worsen
hyperkalemia. But, beta-2-adrenergic agonists, such as adrenaline, can lower plasma
glucose. Thus, beta-blockers should not be used in treating hypertension in this
patient.
Key point:
In addition to treatment of diabetes mellitus, insulin can also be used in treatment of
sever hyperkalemia.
Question #95
QID: 711
Topic: Primary Polydipsia
Subject: Medicine
An 18-year-old girl presents with thirst and polyuria in the order of 4-5 L per 24
hours. The urinalysis (urine microscopy) reveals no glucose or albumin and a normal
sediment. The urine specific gravity is 1.001, and the serum sodium 137 mmol/L.
After 8 hours with no fluid intake, the urine specific gravity is 1.024 and the serum
sodium 141 mmol/L.
Which one of the following is the most likely diagnosis?
a) Primary polydipsia
b) Chronic renal failure
c) Neurogenic (central) diabetes insipidus
d) Nephrogenic diabetes insipidus
e) Syndrome of inappropriate antidiuretic hormone (ADH) secretion
Explanation:
Primary polydipsia, also termed psychogenic diabetes insipidus, or compulsive water
drinking. It is a condition in which the subject experiences a powerful need to ingest
fluid and presents with polyuria and polydipsia and a plasma osmolarity lower than
the normal, perhaps as low as or lower than 270 mOsm/L. The synthesis, secretion,
nephron sensitivity to ADH and the osmoreceptor response are all intact and operative
in these subjects.
Question #96
QID: 795
Topic: Hyperthyroidism
Subject: Medicine
Explanation:
Hyperthyroidism is characterized by hypermetabolism and elevated serum levels of
free thyroid hormones. Symptoms are many but include tachycardia, fatigue, weight
loss, and tremor.
Diagnosis is based on history, physical examination, and thyroid function tests. Serum
TSH is the best test, because TSH is suppressed in hyperthyroid patients except when
the etiology is a TSH-secreting pituitary adenoma or pituitary resistance to thyroid
hormone. Free T4 is increased.
Question #97
QID: 2903
Topic: Hyperthyroidism
Subject: Medicine
A 27-year-old female complains of palpitations and heat intolerance. She had similar
symptoms before being diagnosed with hyperthyroidism and treated with
propylthiouracil. The propylthiouracil was discontinued 6 weeks ago by an
emergency department physician because the patient developed a rash. The rash
resolved and the patient did not keep a recommended follow-up appointment with
you.
On physical examination, the patient’s pulse is 134 beats/min and regular, her blood
pressure is 136/58 mm Hg, and her temperature is 36.9˚C (98.4˚F). The thyroid is
diffusely enlarged without nodularity. The skin is moist and a fine tremor is present.
The examination is otherwise unremarkable.
Which one of the following is most likely to be effective for initial symptom control?
a) Propranolol (Inderal)
b) Potassium iodide (SSKI)
c) Verapamil (Calan, Isoptin)
d) Adenosine (Adenocard)
e) Diltiazem (Cardizem)
Explanation:
Symptoms of hyperthyroidism including palpitations, heat intolerance, tachycardia,
and tremor are largely mediated by increased adrenergic activity. Propranolol is useful
for treating the symptoms of mild hyperthyroidism. It is effective in decreasing
tachycardia and tremor and also has a role in the treatment of thyroid storm. At high
doses it blocks conversion of T4 to T3, in addition to providing β-blockade.
Antithyroid agents such as propylthiouracil are useful, but require an extended period
for symptom control. Potassium iodide should not be used until antithyroid therapy
with an agent such as propylthiouracil has been instituted. It is most frequently used
as part of the treatment of thyroid storm.
Calcium channel blockers such as diltiazem and verapamil may be useful in slowing
supraventricular arrhythmias by blocking conduction at the AV node. They are not
useful for the adrenergically triggered sinus tachycardia of hyperthyroidism.
Adenosine has similar but transient effects on the AV node.
Question #98
QID: 5509
Topic: Hyperthyroidism
Subject: Medicine
Explanation:
Overt hyperthyroidism causes an increase in neonatal morbidity from preterm birth
and low birth weight. Propylthiouracil should be considered the treatment of choice
because methimazole may be associated with congenital anomalies. 131I is
contraindicated in pregnancy because of radiation dangers to the fetus, as well as
thyroid destruction. Although subtotal thyroidectomy is a viable treatment option, it is
recommended only if medical therapy is unsuccessful.
Question #99
QID: 6164
Topic: Hyperthyroidism
Subject: Medicine
Explanation:
Amiodarone is 37% iodine and is the most common source of iodine excess in
Canada. Excessive iodine intake from dietary sources, radiographic contrast media, or
amiodarone increases the production and release of thyroid hormone in iodine-
deficient individuals and in older persons with multinodular goiter. Additionally, like
other medications such as interferon and interleukin-2, amiodarone can trigger
thyroiditis in patients with normal thyroid glands. These characteristics combine to
induce hyperthyroidism in slightly over 10% of patients treated with amiodarone. -
Blockers such as propranolol may be useful in controlling the symptoms of
hyperthyroidism. Methimazole and propylthiouracil interfere with organification of
iodine, thereby suppressing thyroid hormone production; they are commonly used as
antithyroid agents when treating hyperthyroidism. Research is ongoing to determine if
methotrexate plus prednisone is an effective treatment for the ophthalmopathy
associated with Graves’ hyperthyroidism.
Question #100
QID: 10885
Topic: Hyperthyroidism
Subject: Medicine
Four weeks ago a patient presented to her family physician with palpitations and heat
intolerance. With proper work-up, she was diagnosed as a case of Grave’s Disease.
She was treated accordingly. Today, she comes back for follow-up. Adjustment of the
dose of her antithyroid medication should be based on the serum level of:
a) TSH
b) Total thyroxine (T4)
c) Total triiodothyronine (T3)
d) Free thyroxine (T4)
e) Free triiodothyronine (T3)
Explanation:
Serum free T4 is the test of choice for titration of the dose of antithyroid medications
because of its high sensitivity to the activity of the thyroid gland. This high sensitivity
reflects the fact that T4 is the major product of the gland. T4 represents the biologically
active form of thyroxine and is the source of most circulating T3.
Key point:
Although serum TSH is the most sensitive test for diagnosing hyperthryoidism, it is
insensitive for evaluating the effect of treatment and titration of the dose of
antithyroid medications. Free T4 should be used instead.
Question #101
QID: 866
Topic: FSH
Subject: Medicine
Low to normal follicle-stimulating hormone levels (FSH) are found in patients with:
a) Resistant ovary syndrome
b) Previous pelvic radiotherapy for cervical cancer
c) Anorexia nervosa
d) Pure gonadal dysgenesis
e) Premature ovarian failure
Explanation:
A low caloric intake not only interferes with the nutritional needs of a young and
growing organism, but also interferes with the homeostatic mechanisms necessary to
maintain functioning. Undernutrition and weight loss associated with anorexia
nervosa either reverse pubertal changes and prevent menarche from occurring
(primary amenorrhea) or regress the hypothalamic-pituitary-gonadal axis to
prepubertal regulation, resulting in low LH, low FSH and minimal estrogen and
progesterone plasma levels (secondary amenorhhea).
Question #102
QID: 1099
Topic: Hypokalemia
Subject: Medicine
A 62 year old woman who is a regular patient calls the office because she has
developed severe muscle weakness, muscle cramps and polyuria. She began treatment
6 weeks ago with 50 mg of chlorthalidone daily for mild-to-moderate essential
hypertension.
The most likely explanation for her symptoms is the development of which of the
following?
a) Diabetes mellitus
b) Hypokalemia
c) Hypomagnesemia
d) Hyponatremia
e) Metabolic acidosis
Explanation:
Hypokalemia is a very common side effect of nonpotassium-sparing diuretics (e.g.,
chlorthalidone). This is often more pronounced in the older age group. Patients
usually complain of muscle weakness, fatigue, and cramps. Constipation and ileus
characterize the smooth muscle involvement, whereas hyporeflexia, flaccid paralysis,
and tetany are signs of severe hypokalemia.
Question #103
QID: 1139
Topic: Diabetic Ketoacidosis
Subject: Medicine
a) Remain unaffected
b) Can appear normal but total body potassium may actually be low
c) Can appear normal but total body potassium may actually be high
d) Will naturally be corrected by insulin administration
e) None of the above
Explanation:
In the course of DKA, serum potassium levels can appear normal but total body
potassium may actually be low.
Question #104
QID: 1154
Topic: Hyperlipidemia
Subject: Medicine
A 51 year old woman presents to her physician for follow up of a fasting serum
cholesterol level of 13.5 mmol/L. She is post-menopausal since age 49, and has not
been on hormone replacement therapy. She has a positive family history for coronary
artery disease and she has smoked one-half pack of cigarettes per day for the past 15
years. Today she would like to know the results of her full lipid panel.
Which of the following lipid panels would most strongly suggest the need for
pharmacologic therapy in this patient?
Question #105
QID: 2017
Topic: Hyperlipidemia
Subject: Medicine
a) Hyperthyroidism
b) Hypothyroidism
c) Addison’s disease
d) Cushing’s disease
e) Pernicious anemia
Explanation:
According to the Summary of the National Cholesterol Education Program (NCEP)
Adult Treatment Panel III Report, any person with elevated LDL cholesterol or any
other form of hyperlipidemia should undergo clinical or laboratory assessment to rule
out secondary dyslipidemia before initiation of lipid-lowering therapy. Causes of
secondary dyslipidemia include diabetes mellitus, hypothyroidism, obstructive liver
disease, chronic renal failure, and some medications.
Question #106
QID: 3209
Topic: Hyperlipidemia
Subject: Medicine
A mildly hypertensive 54-year-old male with type 2 diabetes asks about the benefit of
adding fish oil capsules to his daily medication regimen of atorvastatin (Lipitor),
metformin (Glucophage), and aspirin.
You advise that fish oil supplements have been shown to:
a) increase the risk of rhabdomyolysis
b) lower triglycerides
c) decrease cardiovascular risks in the general population
d) have antiarrhythmic properties
e) improve glycemic control
Explanation:
The omega-3 polyunsaturated fatty acids found in fish oil have been shown to lower
plasma triglycerides and reduce the risk of stroke after a myocardial infarction.
Unlike niacin and gemfibrozil, they do not increase the risk of rhabdomyolysis in
statin patients. However, they may worsen glycemic control in diabetics and have a
proarrhythmic effect in coronary artery disease patients. There is no evidence that fish
oil supplements prevent coronary disease in the general population.
Question #107
QID: 6237
Topic: Hyperlipidemia
Subject: Medicine
Explanation:
According to the National Cholesterol Education Program Adult Treatment Panel III
Report of 2001, any person with elevated LDL cholesterol or any other form of
hyperlipidemia should undergo clinical or laboratory assessment to rule out secondary
dyslipidemia before initiation of lipid-lowering therapy. Causes of secondary
dyslipidemia include diabetes mellitus, hypothyroidism, obstructive liver disease,
chronic renal failure, and some medications.
Question #108
QID: 1196
Topic: Thyroid Disease
Subject: Medicine
a) Neurofibromatosis
b) Vitiligo
c) Erythema nodosum
d) Pemphigus vulgaris
e) Icthyosis vulgaris
Explanation:
Vitiligo is a loss of skin melanocytes that causes areas of skin depigmentation of
varying sizes. Cause is unknown, but the condition may be autoimmune, as up to 1/3
of patients have evidence of other autoimmune disease. Vitiligo affects 0.5 to 2% of
the population. Etiology is unknown, but melanocytes are lacking in affected areas;
some patients have antibodies to melanin. Up to 30% have other autoimmune
antibodies (to thyroglobulin, adrenal cells, and parietal cells) or clinical autoimmune
endocrinopathies (Addison's disease, diabetes mellitus, pernicious anemia, and
thyroid dysfunction), leading to speculation that vitiligo is an autoimmune disease.
However, the relationship is unclear and may be coincidental. The strongest
association is with hyperthyroidism (Graves' disease) and hypothyroidism
(Hashimoto's thyroiditis).
Question #109
QID: 2037
Topic: Thyroid Disease
Subject: Medicine
Explanation:
Even mild chronic excess thyroid hormone replacement over many years can cause
bone mineral resorption, increase serum calcium levels, and lead to osteoporosis.
Question #110
QID: 3341
Topic: Thyroid Disease
Subject: Medicine
Which one of the following is the most accurate and cost-effective screening
procedure for thyroid disease in geriatric patients?
a) Serum T3
b) Serum free T4
c) Radioactive iodine (RAIU) uptake
d) Serum TSH
e) Radioallergosorbent test (RAST)
Explanation:
Physicians should consider periodic screening for biochemical abnormalities in
elderly patients, because a clinical presentation of thyroid disease is not reliable in this
age group. The most effective way to screen for thyroid dysfunction is to order a TSH
level with third-generation assays using an immunoradiometric assay (IRMA). The
RAST test is used for allergy testing. Notably, the College of Physicians recommends
periodic screening only for women age 50 and older. Other organizations have
varying guidelines.
Question #111
QID: 1212
Topic: Graves Disease
Subject: Medicine
In advising her about the prognosis, the most accurate statement is:
Explanation:
Hyperthyroid patients are at increased risk for cardiac arrhythmias, typically
tachyarrhythmias. Graves’ ophthalmopathy follows a course that is independent of
thyroid disease. Malignant degeneration is not a complication of Graves disease.
Hyperthyroidism does not preclude the ability to become pregnant. There is no
evidence to suggest that every Graves thyroid increases in size without surgery.
Question #112
QID: 5640
Topic: Graves Disease
Subject: Medicine
A 33-year-old white female presents with tremor and a history of weight loss. On
examination she is found to have mild, regular tachycardia and exophthalmos.
Laboratory tests confirm hyperthyroidism. Which one of the following treatments has
been found to potentially worsen ophthalmopathy?
a) Radioactive iodine
b) Propylthiouracil
c) Methimazole (Tapazole)
d) Thyroid hormone replacement plus propylthiouracil
e) Thyroidectomy
Explanation:
The ophthalmopathy of Graves’ disease may initially flare and worsen when treated
with radioactive iodine. Antithyroid drugs, including carbimazole, propylthiouracil,
and methimazole, are not associated with this problem. The addition of thyroid
hormone to these drugs at suppressive doses has not shown any clear benefit over
titration of the antithyroid drug, and relapse rates are similar. Thyroid surgery in the
controlled patient has not been significantly associated with this problem.
Question #113
QID: 6099
Topic: Graves Disease
Subject: Medicine
In a patient with symptoms of thyrotoxicosis and elevated free thyroxine (T4), and the
presence of thyroid-stimulating thyrotropin receptor antibodies would indicate which
one of the following as the cause of thyroid gland enlargement?
a) Toxic multinodular goiter
b) Toxic adenoma
c) Hashimoto’s (lymphadenoid) thyroiditis
d) Subacute (giant cell) thyroiditis
e) Graves’ disease
Explanation:
Graves’disease is characterized by thyroid stimulating autoantibodies directed against
the thyrotropin receptor leading to receptor activation in the absence of
TSH. Thyroid-stimulating immunoglobulins (TSI) bind to and activate thyrotropin
receptors, causing the thyroid gland to grow and the thyroid follicles to increase
synthesis of thyroid hormone. Detection of TSIs is diagnostic for Graves disease.
Patients usually present with symptoms typical of thyrotoxicosis. Hyperthyroidism is
characterized by both increased sympathetic and decreased vagal modulation.
Tachycardia and palpitation are very common symptoms. Free T4 level is usually
elevated, as is the free T3 level.
> A toxic nodular goiter is a thyroid gland that contains autonomously functioning
thyroid nodules, with resulting hyperthyroidism. It is not associated with autoimmune
thyroiditis.
> A thyroid adenoma is a benign tumor of the thyroid gland. Almost all thyroid
adenomas are follicular adenomas. Toxic thyroid adenoma produces excessive thyroid
hormone but is not associated with autoimmune thyroiditis.
> Hashimoto thyroiditis is part of the spectrum of autoimmune thyroid diseases and is
characterized by the destruction of thyroid cells by various cell- and antibody-
mediated immune processes. Laboratory studies show low total T4 or free T4 level in
the presence of an elevated TSH level and hyroid autoantibodies: typically anti-
thyroid peroxidase and anti-thyroglobulin antibodies.
> Subacute thyroiditis is a self-limited thyroid condition (not associated with
autoimmune thyroiditis) associated with a triphasic clinical course of
hyperthyroidism, hypothyroidism, and return to normal thyroid function. Subacute
thyroiditis may be responsible for 15-20% of patients presenting with thyrotoxicosis
and 10% of patients presenting with hypothyroidism.
Question #114
QID: 10905
Topic: Graves Disease
Subject: Medicine
A 40-year-old female presents because she has noticed a neck mass and complains of
weight loss despite normal appetite. She denies pain, hair loss, and constipation.
Exopthalmos is noted on physical examination. What is the most likely diagnosis?
Explanation:
This patient’s clinical presentation with weight loss, neck mass, and exophthalmos
combined together are suggestive of Graves disease, which is diffuse toxic goiter
(choice C). It is an autoimmune disease characterized by hyperthyroidism due to
circulating autoantibodies. The thyroid gland is under continuous stimulation by
circulating autoantibodies against the thyrotropin receptor, and pituitary thyrotropin
secretion is suppressed because of the increased production of thyroid hormones.
Almost every organ system can be affected by Graves disease. Patients may present
with warm and moist skin, sweating, tremors, palpitations, chest pain, photophobia,
polyuria, heat intolerance, amenorrhea, and irritability. Out of the choices given, no
other diagnosis would have the clinical presentation of this patient.
Key point:
Neck mass, symptoms associated with hyperthyroidism, and exopthalmos are
suggestive of diffuse toxic goiter also known as Graves disease.
Question #115
QID: 1566
Topic: Obesity
Subject: Medicine
When considering therapy for the obese patient, one should know that the major
component of caloric expenditure is:
Explanation:
When caloric intake is greater than energy expenditure, obesity results. The major
components of caloric expenditure are the basal metabolic rate, dietary thermogenesis,
and physical activity. The basal metabolic rate (the amount of energy required to
maintain metabolic homeostasis) accounts for 60%-70% of caloric expenditure.
Dietary thermogenesis (the energy required for digestion and metabolism of food)
accounts for 5%-10% of caloric expenditure and depends upon the amount and type
of food eaten and, to an extent, on the degree of obesity. Physical activity accounts for
about 25%-35% of caloric expenditure in the average person.
Question #116
QID: 1591
Topic: Diagnostic Investigations
Subject: Medicine
Of the following, the best laboratory study to evaluate nutritional status in an elderly
patient is:
a) Folate levels
b) Iron studies
c) Serum creatinine
d) Serum albumin
e) Serum electrolytes
Explanation:
Serum albumin is sensitive to changes in nutritional status, although its long half-life
(17-21 days) means that return to normal is slow. A lymphocyte count provides a
quick estimate of immune status and low counts may indicate a need for nutritional
assessment. Iron studies are more likely to be abnormal as a result of gastrointestinal
blood loss. Folate level mostly reflect generic nutritional deficiency, rather than a
specific deficiency. A low folate level is consistent with consumption of a limited
variety and quantity of food. Electrolyte levels do not reflect nutrition.
Question #117
QID: 1638
Topic: Diagnostic Investigations
Subject: Medicine
a) Secondary hypertension
b) Morbid obesity
c) Cor pulmonale
d) Sickle cell disease
e) Hypothyroidism
Explanation:
The glycosylated hemoglobin assay is rendered inaccurate by conditions affecting red
blood cell survival, such as sickle cell disease or the presence of hemoglobin C.
Question #118
QID: 3169
Topic: Diagnostic Investigations
Subject: Medicine
A 26 year old white female with regular periods presents to your office with
complaints of excessive hair growth. An examination reveals moderately increased
facial and body hair.
Which one of the following studies would you order initially to evaluate androgen
excess?
a) Dehydroepiandrosterone
b) FSH
c) Free testosterone
d) Prolactin
e) 17 alpha-hydroxyprogesterone
Explanation:
With mild hirsutism, regular menses, and no features to suggest a secondary cause, it
is reasonable to forgo laboratory evaluation given the very high likelihood that the
hirsutism is idiopathic. About half of women with moderate or severe hirsutism have
androgen excess. The initial step in the workup should be measurement of plasma free
testosterone in the early morning, ideally between days 4 and 10 of the menstrual
cycle. If free testosterone is abnormal, further workup would include an evaluation for
an underlying cause.
If this woman with hirsutism had reported irregular menses consistent with
anovulation, polycystic ovary syndrome (PCOS) should be suspected as the cause of
her androgen excess and hirsutism. The workup for PCOS should include evaluation
for glucose intolerance and metabolic syndrome.
Question #119
QID: 3284
Topic: Diagnostic Investigations
Subject: Medicine
Which one of the following is true regarding thyroid testing in this situation?
a) Most patients who are critically ill should be tested for occult hypothyroidism
b) Serum TSH is a good screening test for thyroid disease in critically ill patients
c) low TSH level in this patient would indicate hyperthyroidism
d) A low T4 level is a frequent finding in patients hospitalized in the intensive-care
unit
e) A low T3 level should be treated with thyroid replacement in hospitalized critically
ill
patients
Explanation:
Many patients hospitalized with nonthyroid illness in the intensive care unit have
abnormalities on thyroid testing. Low serum concentrations of both thyroxine (T4) and
triiodothyronine (T3) are common, and TSH may be decreased as well. Thyroid
function should not be assessed in seriously ill patients unless a thyroid problem is
strongly suspected. Measurement of serum TSH alone is inadequate for the evaluation
of thyroid function in critically ill patients.
Question #120
QID: 5629
Topic: Diagnostic Investigations
Subject: Medicine
A 45-year-old male was admitted to the hospital for nausea resulting from
chemotherapy for colon cancer. He has no other chronic diseases and takes no routine
medications. He was mildly dehydrated on admission and has been receiving
intravenous fluids (D5 ½-normal saline with potassium chloride) at slightly higher
than maintenance rates through an indwelling port for the last 24 hours. The nausea is
being controlled by antiemetics, and his condition is improving. Results of routine
blood work at the time of admission and from the following morning are shown
below.
Which one of the following would be the most appropriate next step?
a) Start an intravenous insulin drip
b) Order blood work taken from a peripheral vein
c) Restrict the patient’s free water intake
d) Switch from normal saline to hypertonic saline
e) Treat with diuretics
Explanation:
You should avoid reacting to laboratory values without considering the clinical
scenario. This patient presented with mild dehydration and normal laboratory values.
Although he is improving clinically, his laboratory values show multiple unexpected
results. The most noticeable is the severely elevated glucose, because he has no
history of diabetes mellitus or use of medications that could cause this effect.
Similarly, the elevated potassium and decreased sodium suggest profound electrolyte
abnormalities. Most likely, the laboratory technician drew blood from the patient’s
indwelling port without discarding the first several milliliters. Thus, the blood was
contaminated with intravenous fluids, resulting in the erroneous results. A repeat
blood test from a peripheral vein should give more accurate results.
Question #121
QID: 6026
Topic: Diagnostic Investigations
Subject: Medicine
Which one of the following is the best diagnostic test for vitamin D deficiency?
a) Ionized calcium
b) Serum phosphorus
c) 24-hour urine for calcium
d) 1,25-hydroxyvitamin D
e) 25-hydroxyvitamin D
Explanation:
Undiagnosed vitamin D deficiency is not uncommon, and 25-hydroxyvitamin D is the
barometer for vitamin D status. Although there is no consensus on optimal levels of
25-hydroxyvitamin D as measured in serum, vitamin D deficiency is defined by most
experts as a 25-hydroxyvitamin D level of <20 ng/mL (50 nmol/L).
Question #122
QID: 6167
Topic: Diagnostic Investigations
Subject: Medicine
Explanation:
In patients with fatigue, you should complete an appropriate history and physical
examination. Laboratory studies should be considered, although the results affect
management in only 5% of patients. A baseline cortisol level would be valuable only
in patients with significant findings of Addison’s disease. In addition to an
erythrocyte sedimentation rate, a complete metabolic panel, and a TSH level, many
physicians request a CBC and a urinalysis. A pregnancy test should be ordered for
women of childbearing age. No other tests have been shown to be useful unless a
specific medical condition is suspected.
Question #123
QID: 1611
Topic: Hypernatremia
Subject: Medicine
a) Resistance to dehydration
b) A tendency toward hypernatremia due to diminished thirst response
c) Lower than usual concentrations of medications distributed in body fluids
d) Greater than usual required dosages of medications
Explanation:
There is no known resistance to dehydration in the elderly. There is a smaller volume
of dilution for drugs distributed in body fluids. This may result in higher
concentrations of medications, sometimes approaching toxic levels, especially if the
usual adult dosages are given. Because elderly patients have a diminished thirst
response, they are prone to develop hypernatremia when water is not immediately
available.
Question #124
QID: 1779
Topic: Thyrotoxicosis
Subject: Medicine
a) Graves' disease
b) Subacute thyroiditis
c) Toxic multinodular goiter
d) Solitary toxic thyroid nodule
Explanation:
Thyrotoxicosis with a high 24-hours radioactive iodine uptake (RAIU) is caused by
Graves’ disease, toxic multinodular goiter, a solitary hot nodule, a TSH-secreting
pituitary tumor, molar pregnancy, and choriocarcinoma. Thyrotoxicosis with a low
24-hour RAIU may be the result of subacute thyroiditis, sporadic silent thyroiditis,
postpartum lymphocyctic thyroiditis, radiation-induced thyroiditis, iodine-induced
thyroiditis, thyrotoxicosis factita, metastatic follicular thyroid cancer, and struma
ovarii.
Question #125
QID: 1904
Topic: Adverse Drug Effect
Subject: Medicine
Which one of the following is more likely to occur with glipizide (Glucotrol) than
with metformin (Glucophage)?
a) Lactic acidosis
b) Hypoglycemia
c) Weight loss
d) Gastrointestinal distress
Explanation:
Metformin is a biguanide used as an oral antidiabetic agent. One of its main
advantages over some other oral agents is that it does not cause hypoglycemia. Lactic
acidosis, while rare, can occur in patients with renal impairment. In contrast to most
other agents for the control of elevated glucose, which often cause weight gain,
metformin reduces insulin levels and more frequently has a weight-maintaining or
even a weight loss effect. Gastrointestinal distress is a common side-effect of
metformin, particularly early in therapy.
Question #126
QID: 2879
Topic: Adverse Drug Effect
Subject: Medicine
When used to lower total serum cholesterol, nicotinic acid tends to:
Explanation:
Nicotinic acid lowers total serum cholesterol, LDL cholesterol, serum triglycerides,
and the LDL/HDL ratio. It increase HDL cholesterol. It worsens glucose control, and
is therefore not a good choice for use in diabetics.
Question #127
QID: 2924
Topic: Adverse Drug Effect
Subject: Medicine
Which one of the following forms of glucocorticoid therapy is most likely to result in
hypothalamic-pituitary-adrenal axis suppression?
Explanation:
Late evening doses of glucocorticoid are more likely to suppress the normal early
morning ACTH surge. They should be avoided because of the increased risk of
suppressed adrenal secretion.
Question #128
QID: 2957
Topic: Adverse Drug Effect
Subject: Medicine
A 62-year-old white male has a history of coronary artery disease, hypertension, and
osteoarthritis. He is on lovastatin (Mevacor) for hypercholesterolemia. You are
choosing a medication to treat his hypertension.
Which one of the following could increase this patient’s risk of rhabdomyolysis?
a) Carvedilol (Coreg)
b) Quinapril (Accupril)
c) Losartan (Cozaar)
d) Atenolol (Tenormin)
e) Verapamil (Calan, Isoptin)
Explanation:
Lovastatin, as well as other statin cholesterol-lowering medications, is primarily
metabolized through the cytochrome P-450 system. Medications that inhibit the
enzyme CYP 3A4, such as verapamil, increase serum concentrations of selected
statins and subsequently increase the risk of rhabdomyolysis.
Question #129
QID: 3175
Topic: Adverse Drug Effect
Subject: Medicine
A 67 year old male presents with symptoms of decreased libido and erectile
dysfunction. An evaluation reveals that he has primary testosterone deficiency, and
you consider testosterone replacement therapy.
Explanation:
When testosterone is taken at usual physiologic doses, liver toxicity is unusual.
Testosterone replacement therapy may stop bone loss and increase bone density, but
many studies have been equivocal, and none have reported a decreased rate of
fractures. Although prostate cancer is thought to be stimulated by testosterone, no
studies have shown convincing evidence of an increased risk of prostate cancer
associated with testosterone therapy. Studies indicate that testosterone therapy either
has no effect on HDL cholesterol or reduces HDL cholesterol levels. Testosterone
stimulates erythropoiesis in 3%-18% of men receiving testosterone therapy. Since
polycythemia is a potential issue, hemoglobin levels should be monitored.
Question #130
QID: 5423
Topic: Adverse Drug Effect
Subject: Medicine
The symptoms started 3 weeks ago. She also complains of weight loss, increased
appetite and heat intolerance. Her vital signs are within normal limits except for a
pulse of 100/min. Physical exam shows that her thyroid gland is diffusely enlarged.
You order thyroid function tests which show low TSH, high T4 and T3. After proper
counseling, the patient refuses surgery or radioactive iodine ablation. You prescribed
propylthiouracil for her. Which of the following is the most common potentially life
threatening side effect of this medication?
a) Hypocalcemia
b) Liver failure
c) Renal failure
d) Serious infection
e) Stomach cancer
Explanation:
No blood! No Drug! Patients taking propylthiouracil for a long time should be
followed routinely with blood tests “CBC with differential” to guard against the
potentially life threatening agranulocytosis which could put the patient at risk for
disseminated infections.
Liver necrosis and failure is a possible side effect for propylthiouracil; however it is
very rare to occur. Always read the statement of the question carefully; we are asking
for the most common one. Do not let them fool you on the exam!
Renal failure and stomach cancer are not associated with propylthiouracil.
Question #131
QID: 5543
Topic: Adverse Drug Effect
Subject: Medicine
Explanation:
Testosterone increases hematocrit and can cause polycythemia. In patients receiving
testosterone supplementation, hematocrit should be monitored every 6 months for the
first 18 months, then annually. Testosterone should be discontinued if there is more
than a 50% rise in hematocrit. Testosterone also causes an increase in lean body mass,
and may increase bone density and increase PSA. No major unfavorable effects on
lipids were reported.
Question #132
QID: 6080
Topic: Adverse Drug Effect
Subject: Medicine
Which one of the following decreases the absorption of orally administered calcium
supplements?
a) Taking calcium carbonate with meals
b) Taking calcium citrate with meals
c) Vitamin D supplementation
d) Proton pump inhibitors
Explanation:
Long-term histamine H -blocker or proton pump inhibitor use is associated with
decreased absorption of 2 calcium carbonate. Patients taking these medications who
require calcium supplementation should use calcium citrate to improve absorption.
Calcium carbonate preparations should be given with a meal to improve absorption.
Vitamin D is important in calcium absorption.
Question #133
QID: 6165
Topic: Adverse Drug Effect
Subject: Medicine
Explanation:
Dehydroepiandrosterone (DHEA) is illegal under the Anabolic Steroid Control Act of
2004, and is prohibited by the NCAA and the International Olympic Committee. Like
androstenedione, DHEA is a precursor to testosterone, but neither of these substances
has been shown to enhance either performance or strength. In fact, they increase
serum estrogen and luteinizing hormone levels.
Question #134
QID: 2976
Topic: Insulin Therapy
Subject: Medicine
For which one of the following insulins does the onset of action occur within 15
minutes, with peak action occurring within about 1 hour?
a) NPH
b) Regular insulin (Novolin R, Humulin R)
c) Lispro (Humalog)
d) Lente (insulin zinc suspension)
e) Glargine (Lantus)
Explanation:
The traditional bolus or mealtime insulin is regular insulin that has an onset of action
within 30-60 minutes and peaks in 2-3 hours. As a result, regular insulin must be
injected 30-60 minutes before meals. In order to avoid this problem, analogue insulins
have been created by substituting 1-3 amino acids in the human insulin protein, which
allows for altered absorption rates and more reliable absorption profiles. The analogue
forms of bolus or mealtime insulin include lispro and asparte. Both of these begin
acting within 15 minutes and peak in about 1 hour, mimicking normal mealtime
insulin release. NPH, lente, and glargine are all basal insulins with a longer onset,
peak, and duration of action.
Question #135
QID: 2872
Topic: Hypophosphatemia
Subject: Medicine
An elderly alcoholic male is brought to the hospital by his grandson, who found him
in poor condition. The grandson reports that his family has not seen the patient in
months.
The patient denies complaints, but is obviously malnourished, poorly washed, and
mildly intoxicated. You admit the patient to the hospital and begin providing
supportive care, including intravenous fluids with dextrose, a regular diet, and
physical therapy evaluation. On the evening of the second day he becomes weak and
more confused. His blood pressure is 88/56 mm Hg, and he has a seizure. Your
evaluation includes the following laboratory findings:
This episode is most likely related to abnormal levels of which one of the following?
a) Glucose
b) Creatinine
c) Creatine kinase
d) Calcium
e) Phosphate
Explanation:
Severe hypophosphatemia is a medical emergency. In poorly nourished patients,
refeeding syndrome can occur. Symptoms usually occur by the second or third day of
improved nutrition, and are often multisystemic. Findings may include weakness,
confusion, dysrhythmias, respiratory failure, congestive heart failure, hypotension,
ileus, metabolic acidosis, seizures, coma, and sudden death. This constellation of
problems results from decreased insulin secretion as stores of intracellular phosphate
become depleted. Providing carbohydrates through intravenous fluids or refeeding
increases insulin secretion, which stimulates cells to take up phosphate, causing
severe hypophosphatemia. In this setting cells are unable to produce enough 2,3
diphosphoglycerate and adenosine triphosphate to meet metabolic demands
While hypoglycemia is another medical emergency, this patient’s glucose level is not
low enough to cause these symptoms. Similarly, renal failure of some type is present,
as is an elevated creatinine kinase suggesting rhabdomyolysis; however, neither of
these problems would be expected to cause this patient’s symptoms. Hypocalcemia
can cause multisystemic problems, including weakness and seizures, but this level is
not critically low and is not associated with hypotension.
Question #136
QID: 3277
Topic: Hypophosphatemia
Subject: Medicine
A 31-year-old male with type 1 diabetes mellitus is admitted to the hospital with
diabetic ketoacidosis and pneumonia. After initial treatment in the emergency
department with intravenous fluids and insulin, laboratory tests reveal a serum
phosphate level of 2.1 mg/dL (N 2.5-5.0). He is asymptomatic except for related
pneumonia symptoms.
Which one of the following would be appropriate management of this patient’s low
serum phosphate level?
a) No therapy
b) Oral phosphate replacement, 2.5-3.5 g/day in divided doses
c) Oral phosphate replacement, 2.5-3.5 g/day in divided doses, and oral vitamin D
supplementation, 400-800 IU/day
d) Intravenous phosphate replacement, 0.08-0.16 mmol/kg over 6 hours
Explanation:
Symptomatic hypophosphatemia rarely occurs unless serum phosphate levels are
below 2.0 mg/dL. Serious symptoms, including rhabdomyolysis, do not occur until
serum phosphate concentrations fall below 1.0 mg/dL. Thus, treatment of
hypophosphatemia with phosphate levels greater than or equal to 2.0 mg/dL is
targeted at an underlying etiology.
> Phosphate supplementation (choice B and choice C) in this setting has not been
shown to be beneficial.
> Intravenous administration (choice D) of phosphate can be dangerous, resulting in
the precipitation of calcium and producing the adverse effects of hypocalcemia, renal
failure, and possibly fatal arrhythmias.
Question #137
QID: 2920
Topic: Metabolic Syndrome
Subject: Medicine
Explanation:
The metabolic syndrome is a combination of obesity, lipid abnormalities,
hypertension, and hyperglycemia. It confers a high risk for cardiovascular disease.
Primary therapy includes exercise, weight loss, and dietary changes. This improves all
aspects of the metabolic syndrome, mainly as a result of decreased insulin resistance.
Antioxidants have no proven benefit. Thyroid hormone has no effect on metabolic
syndrome unless a person is hypothyroid. Intensive insulin regimens improve
glycemic control, and have no effect on other aspects of the syndrome, and may
contribute to weight gain, which adversely affects the syndrome. Rest and relaxation
may make one feel better psychologically, but are not a mainstay of treatment.
Question #138
QID: 2961
Topic: Metabolic Syndrome
Subject: Medicine
You are counseling a 45-year-old obese male regarding weight loss. The patient has
elevated triglyceride levels, low HDL levels, and stage 1 hypertension. He does not
currently take medications and would like to avoid taking medications in the future.
The patient has heard good things about low-carbohydrate diets and asks your opinion
about beginning such a plan.
Explanation:
The recent public interest in low-carbohydrate diets has led to research into the
potential health benefits or risks. Emerging data are mostly encouraging, in that these
diets do not seem to cause the expected increases in blood pressure, LDL levels, or
triglyceride levels that the medical community had first assumed. In fact, these diets
seem to consistently result in more weight loss than low-fat diets, and lead to a more
significant reduction in triglyceride levels. This patient has symptoms of the
metabolic syndrome and has a higher risk of glucose intolerance or diabetes. Low-
carbohydrate diets have been shown to reduce insulin resistance at least as well as, if
not better than, traditional diet plans. More research will likely elicit any long-term
effects of these diets that are not currently known.
Question #139
QID: 3134
Topic: Metabolic Syndrome
Subject: Medicine
A 54 year old female diabetic who takes fenofibrate has the following fasting lipid
levels: total cholesterol 5.18 mmol/L (200 mg/dL), LDL cholesterol 1.66 mmol/L (64
mg/dL), HDL cholesterol 1.0 mmol/L (40 mg/dL), triglycerides 13.6 mmol/L (525
mg/dL). Her weight has increased by 3kg (7lb). She occasionally checks her blood
glucose and says it is “okay.” She admits to occasional diarrhea.
In order to improve this patient’s lipid status, the most appropriate next step is to:
a) add ezetimibe
b) check the patient for Celiac disease
c) order TSH and hemoglobin A1c levels
d) discontinue fenofibrate and prescribe rosuvastatin
Explanation:
Poorly controlled diabetes mellitus and hypothyroidism are often associated with a
metabolic syndrome that can include a mixed dyslipidemia. Ezetimibe and
rosuvastatin will lower the LDL-cholesterol level but have little or no effect on
triglycerides. Celiac disease is not a cause of hypertriglyceridemia, but it may be
associated with malabsorption, leading to hypocholesterolemia.
Question #140
QID: 5582
Topic: Metabolic Syndrome
Subject: Medicine
A 55-year-old white male sees you for follow-up after a recent lipid panel revealed no
improvement in his hyperlipidemia. His total cholesterol level is 7.15 mmol/L, with
an LDL-cholesterol level of 4.7 mmol/L, an HDL-cholesterol level of 0.98 mmol/L,
and a triglyceride level of 7.15 mmol/L. These numbers are similar to two previous
lipid panels obtained over the last several months, despite attempts at lifestyle
changes. He has adequately treated essential hypertension, with a blood pressure of
125/83 mm Hg. There is no history of diabetes mellitus or tobacco use, and no family
history of premature coronary heart disease.
After discussion with the patient, you decide to start prescription drug therapy. The
initial target of this therapy should be to reach his goal level of :
a) LDL cholesterol
b) HDL cholesterol
c) non-HDL cholesterol
d) triglycerides
e) fasting blood glucose
Explanation:
This patient meets the criteria for metabolic syndrome. In addition to lifestyle
changes, pharmacologic treatment for his hyperlipidemia should be considered. The
initial goal of this therapy should be to reach his LDL-cholesterol goal, usually using
a statin. After achievement of this goal, non-HDL cholesterol is the secondary target
for therapy. Non-HDL cholesterol is calculated by subtracting HDL cholesterol from
total cholesterol.
Question #141
QID: 2946
Topic: Subacute Thyroiditis
Subject: Medicine
A 40-year-old white female complains of discomfort in her anterior neck. She also
gives a history of malaise, low-grade fever, and a tender thyroid gland.
Laboratory Findings
WBCs……………………………………..
…............12,100/mm3 (N 4300-10.800)
Granulocytes……………………..
…........................30% (N 42-75)
Monocytes…………………………...........................4% (N 2-9)
Lymphocytes……………………...........
…................66% (N 20-51)
Free thyroxine (FT4)…………..
………....................2.8 ng/dL (N 0.8-2.4)
Erythrocyte sedimendation rate (Westergren)......60
mm/hr (N 0-20)
Which one of the following would most likely be seen on a thyroid nuclear medicine
study?
Explanation:
This patient has a clinical presentation typical of subacute thyroiditis. An elevated
erythrocyte sedimentation rate is an almost certain feature, and slight leukocytosis
may also be seen, as well as a modest degree of thyrototoxicosis and a slightly
elevated serum T4 level. A small thyroid gland would be expected on a thyroid scan.
Because the disease interferes with iodine metabolism, radioiodine uptake is
decreased. Hot, toxic nodules or a multinodular goiter would not be expected.
Generalized thyroid enlargement and increased radioactive iodine uptake would be
expected with Graves’ disease.
Question #142
QID: 3150
Topic: Subacute Thyroiditis
Subject: Medicine
A 45 year old Hispanic female presents with diffuse, tender swelling in her anterior
neck. These symptoms were preceded by a prodrome of sore throat, low-grade fever,
myalgias, and fatigue. On examination she has a tender, enlarged thyroid gland.
Laboratory tests reveal elevated free T4 and low TSH.
Explanation:
Subacute granulomatous thyroiditis is the most common cause of thyroid pain. It is
four times more common in women, most often occurs at 40-50 years of age, and is
usually attributed to a viral infection. Symptoms and signs of this disorder include a
prodrome of myalgias, pharyngitis, low-grade fever, and fatigue, followed by a
tender, diffuse goiter and neck pain that often radiates up to the ear. Hyperthyroidism
is seen in half of affected individuals, and must be differentiated from that found with
Graves’ disease to determine appropriate treatment. The hallmarks of subacute
granulomatous thyroiditis are a very high erythrocyte sedimentation rate (ESR), often
as high as 60-100, and a radioiodine uptake of less than 1% at 24 hours (whereas in
those with Graves’ disease it is elevated). Treatment consists of relieving the thyroid
pain and tenderness with NSAIDs. If no improvement occurs in a week, prednisone
may be given in a dosage of 40-60 mg/day, tapered off over 4-6 weeks.
Question #143
QID: 6152
Topic: Subacute Thyroiditis
Subject: Medicine
A 45-year-old female had myalgias, a sore throat, and a fever 2 weeks ago. She now
has anterior neck tenderness and swelling, with pain radiating up to her ears. An
examination reveals a tender goiter.
Explanation:
Subacute granulomatous thyroiditis is the most common cause of thyroid pain. Free T4
is elevated early in the disease, as it is in Graves’ disease; however, later in the
disease T4 becomes depressed and then returns to normal as the disease resolves.
Pretibial myxedema, exophthalmos, and a thyroid thrill or bruit can all be found in
Graves’ disease, but not in subacute granulomatous thyroiditis. Patients with subacute
granulomatous thyroiditis will have a low radioactive iodine uptake (RAIU) at 24
hours, but patients with Graves’ disease will have an elevated RAIU.
Question #144
QID: 3243
Topic: Hyperandrogenism
Subject: Medicine
a) Acromegaly
b) Carcinoma of the ovary
c) Congenital adrenal hyperplasia
d) Cushing’s syndrome
e) Polycystic ovary syndrome
Explanation:
Hyperandrogenism is caused by polycystic ovary syndrome in at least 75% of cases.
This diagnosis is made when there is otherwise unexplained chronic
hyperandrogenism and oligo-ovulation or anovulation. Women with polycystic ovary
syndrome should also be evaluated for glucose intolerance and the metabolic
syndrome.
Question #145
QID: 3283
Topic: Hypomagnesemia
Subject: Medicine
a) Aldosterone
b) Magnesium
c) Sodium
d) Hydrogen ions
e) Calcium
Explanation:
Approximately half of hospitalized patients in whom serum electrolytes are ordered
have unrecognized hypomagnesemia. Many of these patients have hypokalemia
and/or hypocalcemia. Hypomagnesemia also causes renal potassium wasting, which is
refractory to potassium replacement until magnesium is replaced.
Question #146
QID: 3320
Topic: Glycemic Index
Subject: Medicine
Explanation:
The higher the glycemic index of a food, the more rapidly its carbohydrates are
released into the bloodstream as glucose. Foods with a high glycemic index induce a
more rapid insulin response, are more likely to lead to reactive hypoglycemia and a
metabolic syndrome, and lead to higher levels of circulating insulin.
Question #147
QID: 5886
Topic: Conn's Disease
Subject: Medicine
A 34 year old man is found to have a blood pressure of 180/105 mmHg. He admits to stress at work
and states that his father was diagnosed as hypertensive at the age of 60 but, thus far, has not required
treatment. He denies taking any medications. He smokes 25 cigarettes per day but takes no alcohol. On
physical examination he is not obese, has no signs of cardiovascular disease, and no hypertensive
retinopathy. The results of investigations are given below:
Urine analysis negative, Na 146 mmol/L, K 2.8 mmol/L, Cl 102 mmol/L, HCO3 33 mmol/L, Urea 4.2
mmol/L, Creatinine 92 mmol/L, Glucose 6.2 mmol/L
Explanation:
In primary hyperaldosteronism (Conn’s disease), overproduction of aldosterone leads
to fluid retention and increased blood pressure, weakness, and, rarely, periods of
paralysis. Hyperaldosteronism can be caused by a tumor in the adrenal gland or may
be a response to some diseases. High aldosterone levels can cause high blood pressure
and low potassium levels; low potassium levels may cause weakness, tingling, muscle
spasms, and periods of temporary paralysis. Doctors measure the levels of sodium,
potassium, and aldosterone in the blood. Sometimes, a tumor is removed, or people
take drugs that block the action of aldosterone.
Aldosterone, a hormone produced and secreted by the adrenal glands, signals the
kidneys to excrete less sodium and more potassium. Aldosterone production is
regulated partly by corticotropin (secreted by the pituitary gland) and partly through
the renin-angiotensin-aldosterone system.
Hyperaldosteronism can be caused by a tumor (usually a noncancerous adenoma) in
the adrenal gland (a condition called Conn's syndrome), although sometimes both
glands are involved and are overactive.
Question #148
QID: 8709
Topic: Androgen Insensitivity Syndrome
Subject: Medicine
All of the following features have been described in women with androgen
insensitivity syndrome, except:
a) Mammary aplasia
b) Female phenotype
c) Absence of wolffian duct structures
d) Short vagina
e) Absence of mullerian duct structures
Explanation:
Androgen insensitivity syndrome (AIS) is when a person who is genetically male (has
one X and one Y chromosome) is resistant to male hormones called androgens. As a
result, the person has some or all of the physical characteristics of a woman, despite
having the genetic makeup of a man.
In its classic form (complete androgen resistance), the person appears to be female but
has no uterus, and has sparse armpit and pubic hair. At puberty, female secondary sex
characteristics (e.g., breasts) develop, but menstruation and fertility do not.
Incomplete AIS, however, is often discovered during childhood because a person may
have both male and female physical characteristics. Many have partial fusion of the
outer vaginal lips, an enlarged clitoris, and a short, blind-ending vagina.
The Müllerian ducts will develop to form the fallopian tubes, uterus, and the upper
portion of the vagina. The Wolffian duct develops into a system of connected organs
between the testis and the prostate, the epididymis, the vas deferens, the seminal
vesicle, and the prostate. Both of these ducts will be absent in AIS.
Question #149
QID: 8711
Topic: Androgen Insensitivity Syndrome
Subject: Medicine
a) Genotype is XY
b) Lack of breast development and absence of uterus
c) Serum testosterone is below normal male range
d) Secondary sex characteristics are male
e) The condition is autosomal recessive
Explanation:
Androgen insensitivity syndrome (AIS) is when a person who is genetically male (has
one X and one Y chromosome) is resistant to androgens. As a result, the person has
some or all of the physical characteristics of a woman, despite having the genetic
makeup of a man.
In its classic form (complete androgen resistance), the person appears to be female but
has no uterus, and has sparse armpit and pubic hair. At puberty, female secondary sex
characteristics (e.g., breasts) develop, but menstruation and fertility do not.
Incomplete AIS, however, is often discovered during childhood because a person may
have both male and female physical characteristics. Many have partial fusion of the
outer vaginal lips, an enlarged clitoris, and a short, blind-ending vagina.
Question #150
QID: 8710
Topic: Secondary Amenorrhea
Subject: Medicine
a) Turner syndrome
b) Anorexia
c) Androgen insensitivity syndrome
d) Gonadal dysgenesis
e) Imperforate hymen
Explanation:
Secondary amenorrhea is the absence of menstrual periods for 6 months in a woman
who had previously been regular, or for 12 months in a woman who had irregular
periods. This problem is seen in about 1% of women of reproductive age.
Primary amenorrhea is when the woman has never had a period in her life. This page
will not discuss primary amenorrhea which is rare. The causes of primary amenorrhea
are also quite different from secondary amenorrhea.
Question #151
QID: 8736
Topic: Human Chorionic Gonadotropin
Subject: Medicine
Explanation:
Gestational trophoblastic disease like Hydatidiform moles ("molar pregnancy") or
Choriocarcinoma may produce high levels of ßhCG (due to the presence of
syncytialtrophoblasts- part of the villi that make up the placenta) despite the absence
of an embryo. This, as well as several other conditions (seminoma, germ cell tumors,
teratoma with elements of choriocarcinoma (this is rare), and islet cell tumor), can
lead to elevated hCG readings in the absence of pregnancy.
Question #152
QID: 10349
Topic: Euthyroid Sick Syndrome
Subject: Medicine
A 62-year-old male is admitted to the hospital for urosepsis. His medical history is
significant only for hypertension. On examination he has a temperature of 36.5°C
(97.7°F), a TSH level of 0.2 ₃U/mL (N 0.4-5.0), and a free T4 level of 0.4 ng/dL (N
0.6-1.5).
Which one of the following is the most likely explanation for these findings?
a) Pituitary adenoma
b) Graves’ disease
c) Subacute thyroiditis
d) Subclinical hypothyroidism
e) Euthyroid sick syndrome
Explanation:
The euthyroid sick syndrome refers to alterations in thyroid function tests seen
frequently in hospitalized patients, and decreased thyroid function tests may be seen
early in sepsis. These changes are statistically much more likely to be secondary to
the euthyroid sick syndrome than to unrecognized pituitary or hypothalamic disease.
Graves’ disease generally is a hyperthyroid condition associated with low TSH and
elevated free T4. Subclinical hypothyroidism is diagnosed by high TSH and normal
free T 4 levels.
Subacute thyroiditis most often is a hyperthyroid condition.
Question #153
QID: 11056
Topic: Euthyroid Sick Syndrome
Subject: Medicine
An inpatient with multiple injuries sustained in car accident felt cold last night and
was seen by a family medicine resident who among other investigations ordered
thyroid function tests, which showed low serum total and free T3 level, normal total
and free T4, and normal TSH level.
Which one of the following management options would be best for this patient?
Explanation:
Low serum T3 with normal T4 and TSH in patients with severe diseases or trauma is
the most common presentation of the sick euthyroid syndrome. In this syndrome,
thyroid function is normal (euthyroid) as indicated by normal serum TSH. These
patients need no more than observation and monitoring during recovery (choice E).
Indeed, without strong suspicion of thyroid disorders, it is not advisable to perform
thyroid function tests in patients with acute severe diseases or trauma.
> Starting injection of liothyronine (choice A) or thyroxine (choice B) are not the
correct choices. In the absence of history or physical signs suggestive of
hypothyroidism, it is not advisable to give thyroid hormone replacement therapy.
Indeed, according to some reports, thyroid hormone replacement therapy might
worsen the outcome of the diseases causing sick euthyroid syndrome.
> Ordering radioiodine uptake and thyroid scan (choice C) or serum thyroglobulin
level (choice D) are not the correct choices. In the absence of history or physical signs
suggestive of thyroid disease together with the thyroid hormone profile typical of sick
euthyroid syndrome, no further investigation is required.
Key point:
Severe acute disease or trauma may lead to abnormalities of serum thyroid hormone
profile. These disturbance, however, usually resolve spontaneously when patients
recover from the acute disease or trauma.
Question #154
QID: 10831
Topic: Aspartame metabolism
Subject: Medicine
a) Galactosemia
b) Homocystinuria
c) Orotic aciduria
d) Phenylketonuria
Explanation:
Aspartame is completely hydrolyzed in the gut to methanol, aspartic acid, and
phenylalanine, and is therefore contraindicated in patients with phenylketonuria.
Question #155
QID: 10951
Topic: Multiple endocrine neoplasia
Subject: Medicine
A 35-year-old male presents to your department with severe abdominal pain. He had
similar complaints 3 months ago and after evaluation he was prescribed omeprazole
for peptic ulcer disease. He says that the drug seemed to help a bit initially, but
currently he feels his pain is getting worse. He says that the pain is associated with
food. His father died at the age of 51 in a car accident. His mother is 58 years old and
healthy. He has an older brother with a history of kidney stones. On physical
examination he has pain in the epigastric region. Laboratory tests show:
If MEN is suspected, which of the following would be the best choice to arrive at the
correct diagnosis?
a) Abdominal CT scan
b) Endoscopic ultrasound with biopsy
c) 24-hour urine vanillylmandelic acid
d) Head MRI
e) Serum calcitonin
Explanation:
This patient’s abdominal pain is most likely caused by peptic ulcer disease. His
history of abdominal pain, minimal relief with omeprazole, and the laboratory test
results that reveal excessive levels of gastrin point to a gastrinoma (i.e.Zollinger-
Ellison syndrome) as the most likely cause. This patient also has elevated levels of
calcium and a family history of nephrolithiasis, which are suggestive of
hyperparathyroidism. When all of this is taken into account, we must suspect multiple
endocrine neoplasia type I (MEN I). This condition is a constellation of pituitary
adenomas, parathyroid tumors, and pancreatic islet cell tumors.
Pituitary adenomas secrete prolactin in 25% of patients and growth hormone in 5% of
patients. Large pituitary adenomas may cause bitemporal hemianopsia, which is a risk
factor for motor vehicle accidents in these patients. Head MRI (choice D) is
considered the screening test of choice for pituitary adenomas.
Key point:
Multiple endocrine neoplasia type I is a constellation of pituitary adenomas,
parathyroid tumors, and pancreatic islet cell tumors. In a patient who presents with
Zollinger-Ellison syndrome and hypercalcemia, a head MRI is indicated to screen the
patient for pituitary adenoma.
Question #156
QID: 2904
Topic: Hyperglycemia
Subject: Medicine
Laboratory Findings
Sodium………………………………………140
mmol/L (N 135-145)
Potassium……………………………………5.7
mmol/L (N 3.5-4.5)
BUN……………………..……………………65
mg/dL (N 12-20)
Serum creatinine…………...……………….3.2
mg/dL (N 0.6-1.1)
Serum glucose……………………....………36.1
mmol/L
Which of the following is indicated as the first line of management of this patient?
a) Immediate intravenous insulin infusion
b) Oral hypoglycemic agents and training in home glucose testing technique
c) Potassium may be added to the infusion fluid
d) Insulin glargine (Lantus) and insulin lispro (Humalog)
e) Vigorously rehydration with isotonic sodium chloride solution
Explanation:
The patient is in a hyperosmolar, hyperglycemic state (HHS) which is characterized
by hyperglycemia, hyperosmolarity, and dehydration without significant ketoacidosis.
Most patients present with severe dehydration and focal or global neurologic deficits.
This patient should be treated with intravenous fluids and an insulin infusion in an
intensive-care setting, with hourly follow-up of blood glucose levels. Insulin used
without concomitant vigorous fluid replacement increases the risk of shock. Although
she has renal insufficiency and hyperkalemia, the hyperkalemia will resolve with
insulin and fluid infusion. In fact, potassium supplementation will be needed within a
few hours. The delirium will usually clear with correction of the metabolic
abnormalities. Oral diabetic agents or a subcutaneous insulin protocol will be needed
for long-term control, but are not aggressive enough to manage this patient’s acute
problem.
Question #157
QID: 11049
Topic: Acromegaly complication
Subject: Medicine
Which of the following is a particularly common cause of joint pain in a patient with
Acromegaly?
a) Osteoarthritis
b) Overweight
c) Rheumatoid arthritis
d) Iron deposits
e) Gout
Explanation:
The excessive secretion of growth hormone (GH) along with IGF-1 stimulates the
proliferation of cartilage, periarticular connective tissue, and bone, resulting in several
musculoskeletal problems in patients with acromegaly. Even though the options
mentioned could be present in this patient, patients with acromegaly are especially
prone to osteoarthritis (choice A), most often affecting the knees, shoulders, hips and
hands, affecting single or multiple joints.
Hypertrophy of the cartilage initially produces widening of the joint space when seen
on an X-ray. This new cartilage is more susceptible to fissuring, ulceration and
destruction, than normal cartilage. As the degradation of the cartilage progresses, the
joint space narrows, subchondral sclerosis appears, and ultimately, osteophytes are
formed.
> All of the answers are possible causes of this patient's knee pain, yet the most
common is osteoarthritis.